You are on page 1of 40

VISIONIAS

www.visionias.in
ANSWERS & EXPLANATIONS
GENERAL STUDIES (P) TEST – 3195 (2021)

Q 1.B
• Dugongs are vulnerable marine species (IUCN categorized) like sea turtles, seahorses, sea cucumbers
and others. They are protected in India under Schedule I of the Wild Life (Protection) Act, 1972. Hence
statement 3 is correct.
• The dugong (Dugong dugon) is the only herbivorous mammal that is strictly marine. It has a range
spanning some 37 countries, including tropical and subtropical coastal and inland waters. It can be found
in warm coastal waters from East Africa to Australia, including the Red Sea, Indian Ocean, and
Pacific. Hence statement 1 is correct and statement 2 is not correct.
• Commonly known as "sea cows," dugongs graze peacefully on seagrasses in shallow coastal waters.
• These mammals can stay underwater for six minutes before surfacing. They sometimes breathe by
“standing” on their tail with their heads above water.
• Human activities such as the destruction and modification of habitat, pollution, rampant illegal fishing
activities, vessel strikes, unsustainable hunting or poaching, and unplanned tourism are the main threats to
dugongs.
• The loss of seagrass beds due to ocean floor trawling was the most important factor behind dwindling
dugong populations in many parts of the world. Hundreds of dugongs inhabited waters off the Odisha,
West Bengal, and Andhra Pradesh coasts two centuries back. But they are extinct in these areas now.

Q 2.A
• Recent Context – Ministry of Earth Sciences has said that IMD has operationalized Advanced high-
resolution Air Quality Early Warning System for Delhi and India.
• ENFUSER:
o A very high resolution city scale model ENFUSER (ENvironmental information FUsion
SERvice) for Delhi has been operationalized to identify the air pollution hotspots and pollution upto
street level.
o The speciality of the ENFUSER is the high utilization of measurement data such as air quality
observations, a detailed description of the road network, buildings, land-use information, high
resolution satellite images, ground elevation and population data. The ENFUSER natively taps into
the operative IMD’s regional SILAM access point. The ENFUSER results are being evaluated with
the satellite measurements and observations, model is found to capture the hotspots over Delhi very
well. Pollution can be captured better with this model.
• SILAM
o The Air Quality forecast model System for Integrated modelling of Atmospheric composition
(SILAM) for India has been further improved by implementing global emission inventories CAMS-
GLOB v2.1 supplemented with EDGAR v4.3.2 for coarse and mineral-fine anthropogenic particulate
matter at 10km resolution.
• SILAM and ENFUSER have been developed in technical collaboration with Finnish Meteorological
Institute (FMI).
• WRF-Chem
o Air Quality forecast model WRF-Chem has also been updated with high-resolution land use land
cover information to improve the air quality forecast.
• Now AQ early warning system also provides air quality forecast for Lucknow, Kanpur and Varanasi at 2
km resolution. The AQ forecast is also available for some other cities at 10km resolution.
• Hence option (a) is the correct answer.
1 www.visionias.in ©Vision IAS
Q 3.B
• Vernalization is the induction of a plant's flowering process by exposure to the prolonged cold of
winter, or by an artificial equivalent. Hence option (b) is the correct answer.
• After vernalization, plants have acquired the ability to flower. It prevents precocious reproductive
development late in the growing season and enables the plant to have sufficient time to reach maturity.
It shortens the vegetative phase of the plant and helps to increase the fruit set and yield.
• Many plants grown in temperate climates require vernalization and must experience a period of low
winter temperature to initiate or accelerate the flowering process. This ensures that reproductive
development and seed production occurs in spring and winters, rather than in autumn.
• Furthermore, vernalization enhances the plant's resistance to cold temperatures. Vernalization also
increases plant resistance to fungal diseases. And, this technique is one of the best options in horticulture
when grafting a vernalized shoot apex with that of a non-vernalized one. Moreover, vernalization is a
method of crop improvement. It reduces the cost of crop production.
• It is used in the seed treatment of wheat, barley, rye etc. And also subjecting the growing of biennial
plants like sugarbeet, cabbages, carrots to a cold treatment stimulates a subsequent photoperiodic
flowering response.
• Additional Information
o Photoperiodism is the physiological reaction of organisms to the length of the night or a dark
period. It occurs in plants and animals. Photoperiodism can also be defined as the developmental
responses of plants to the relative lengths of light and dark periods.
o Stratification- In horticulture, stratification is a process of treating seeds to simulate natural
conditions that the seeds must experience before germination can occur. Many seed species have an
embryonic dormancy phase and generally will not sprout until this dormancy is broken.
o Seed scarification (a technique to physically damage the seed coat to reduce hard seed while keeping
the seed viable) is used to soften hard seeds. Researchers have been using different scarification
methods since the early 20th century and have reported variable results.

Q 4.B
• Recent Context: A report to the Indian Council for Historical Research (ICHR) has suggested dropping
the Communist martyrs of Punnapra-Vayalar, Karivelloor, and Kavumbayi agitations from the list of
martyrs of India’s Independence struggle.
o C.I. Issac, a member of the ICHR, gave a report to this effect four years ago after scrutinizing the
manuscript of the now controversial publication, “The Dictionary of Martyrs: Freedom Struggle 1857-
1947.”
• About Punnapra Vayalar movement: It was an armed struggle movement against the Maharaja of
Travancore and his Prime Minister, Sir C. P. Ramaswami Iyer, under the leadership of the Communist
Party of India. Hence option (b) is the correct answer.
• The year 1946 witnessed the Dewan of Travancore Sir CP Ramaswamy Iyer declaring the "American
Model" for Travancore princely state, according to which it would remain an independent state. The trade
unions and communist party raised the slogan, “Into the Arabian sea with the American model”. They also
decided to organize agitations to ensure the merger of the princely states with the Indian Union.
Therefore, the workers and peasants of Alappuzha decided to oppose the American model and fight the
police.
• On 24th October 1946, the workers and the peasants fought with the maharaja's army which tried to curb
the movement. The firing took place at Punnapra seashore and hundreds of workers and peasants lost their
lives. The following days there was a fierce fight between the army and the people and more than a
thousand workers lost their lives. This created a new political awareness across the country. There was a
relief committee formed to help the victims of these struggles in Malabar.
• The Punnapra Vayalar struggles served the cause of strengthening the peasant worker unity in the struggle
for agrarian reforms and labor rights. The Punnapra Vayalar struggles gave inspiration to the workers and
peasants of Malabar to carry forward their combined fight against imperialism and landlordism.

Q 5.D
• Manas National Park : It is a national park, UNESCO Natural World Heritage site, a Project Tiger
reserve, an elephant reserve and a biosphere reserve in Assam, India.
o Located in the Himalayan foothills, it is contiguous with the Royal Manas National Park in Bhutan.
o The Manas river flows through the west of the park. It is a major tributary of the Brahmaputra river
and splits into two separate rivers, the Beki and Bholkaduba as it reaches the plains.
2 www.visionias.in ©Vision IAS
o The combination of Sub-Himalayan Bhabar Terai formation along with the riverine succession
continuing up to Sub-Himalayan mountain forest make it one of the richest areas of biodiversity in the
world.
o Fourteen captive-bred pygmy hogs were released in Manas National Park by the Pygmy Hog
Conservation Programme (PHCP). This is a significant milestone in the effort to save one of the
most endangered mammals in the world. Hence pair 1 is correctly matched.
• Dachigam National Park :
o It is located in Jammu and Kashmir. The name literally stands for ‘ten villages’, which could be in
memory of the ten villages that were relocated in order to create the park.
o The park is best known as the home of the hangul, or Kashmir stag. It is categorized as critically
endangered. Hence pair 2 is correctly matched.
• Silent Valley National Park: Located in Palakkad district in Nilgiri Mountains of Kerala.
o It is at the heart of Nilgiri Biosphere Reserve and consists of areas of South Western Ghats rain
forests and tropical wet evergreen jungle. River Kunthi passes through it. Silent Valley Park is
known for many highly endangered species such as lion-tailed macaque, tiger, gaur, leopard, wild
boar, panther, Indian Civet, and Sambhar. Hence pair 3 is correctly matched.

Q 6.D
• The basic objectives of national parks, wildlife sanctuaries, and biosphere reserves are:
o Protection of natural habitats through the controlled, limited use of species.
o Maintenance of the viable number of species in Protected areas.
o Establishment and protection of areas through legislation for the conservation of wildlife.
o Educating the public for wildlife protection.
o Conducting research in specific areas of wildlife. Hence all the given statements are correct.
• A national park is an area that is strictly reserved for the betterment of the wildlife and where human
activities like forestry, grazing or cultivation are not permitted. Example: Jim Corbett National Park,
Sunderbans, etc.
• A wildlife sanctuary is a protected area that is reserved for the conservation only of wildlife animals and
plant species. Human activities like harvesting or timber collection of minor forest products and private
ownership rights are allowed. Example: Periyar Wildlife Sanctuary.
• A biosphere reserve is a special area of land or coastal environment in which multiple uses of land is
permitted by dividing it into certain zones. The natural or core zone consists of an undisturbed and legally
protected ecosystem. The buffer zone surrounds the core area and is managed to accommodate a greater
variety of resource use strategies. The transition zone, the outermost part of the biosphere reserve, is
an area of active cooperation between the management and the local people.

Q 7.D
• Kanha Tiger Reserve: Kanha Tiger Reserve, also known as Kanha–Kisli National Park, is one of the
tiger reserves of India and the largest national park of the state of Madhya Pradesh. It is located in the two
districts Mandla and Balaghat.
o Together with a surrounding buffer zone and the neighbouring Phen Sanctuary, it forms the Kanha
Tiger Reserve, which is one of the biggest in the country.
o The park hosts Bengal tiger, Indian leopard, sloth bear, barasingha and dhole. It is also the first tiger
reserve in India to officially introduce a mascot, Bhoorsingh the Barasingha.
o The area is the ancestral home of the Gond and Baiga tribes. Hence pair 1 is correctly matched.
• Similipal National Park : Similipal National Park is a national park and a tiger reserve in
the Mayurbhanj district of Odisha. It is part of the Mayurbhanj Elephant Reserve, which includes three
protected areas — Similipal Tiger Reserve, Hadgarh Wildlife Sanctuary and Kuldiha Wildlife
Sanctuary. Simlipal National Park derives its name from the abundance of red silk cotton
trees growing in the area.
o The park is home to Bengal tiger, Asian elephant, gaur, and chausingha.
o This protected area is part of the UNESCO World Network of Biosphere Reserves since 2009.
o The high hills surround Meghasani/Tunkiburu, the highest peak in the park.
o At least 12 rivers cut across the plain area. The prominent among them are Budhabalanga, Palpala
Bhandan, Kharkai River and Deo.
o This sprawling forest has two prominent waterfalls - Joranda/Jorodah and Barheipani/Barhai.
o Simlipal comes under a high cerebral malaria-prone zone.
o Similipal forests is home to a variety of tribes. Prominent among these are Kolha, Santhala, Bhumija,
Bhatudi, Gondas, Khadia, Mankadia and Sahara. Hence pair 2 is correctly matched.
3 www.visionias.in ©Vision IAS
• Gir National Park and Wildlife Sanctuary: It is located in the Junagadh district of Gujarat. The Gir
Forests is the only natural habitat of Asiatic lions.
o The Gir Forests is the largest compact tract of dry deciduous forests in the semi-arid western part of
India.
o Gir is often linked with "Maldharis" who have survived through the ages by having a symbiotic
relationship with the lion. Maldharis are religious pastoral communities living in Gir. Their
settlements are called "nesses". Hence pair 3 is not correctly matched.
• Nagarhole National Park: It is a national park located in Kodagu district and Mysore district in
Karnataka. It is one of India's premier Tiger Reserves along with the adjoining Bandipur Tiger Reserve
and Wayanad Wildlife Sanctuary.
o It is part of the Nilgiri Biosphere Reserve. The Western Ghats Nilgiri sub-cluster of 6000km 2,
including all of Nagarhole National Park, is under consideration by the UNESCO World Heritage
Committee for selection as a World Heritage Site.
o The Jenu Kurubasa are main inhabitants of this forest. Hence pair 4 is correctly matched.

Q 8.C
• Millets are a group of small-grained cereal food crops that are highly tolerant of drought and other
extreme weather conditions and are grown with low chemical inputs such as fertilizers and
pesticides. They are rain-fed, hardy grains that have low requirements for water and fertility when
compared to other popular cereals. Hence option 3 is correct.
• Most of the millet crops are native of India and are popularly known as Nutri-cereals as they provide
most of the nutrients required for normal functioning of the human body.
• Millets are classified into Major Millets and Minor Millets based on their grain size. Major
millets are Sorghum (Jowar), Pearl Millet (Bajra), Ragi. Minor millets are Foxtail Millet (Kanngani),
Proso Millet (Cheena), Kodo Millet (Kodo) etc.
• Millets are gluten-free, highly nutritious and non-acid-forming foods. They are rich in micronutrients,
including calcium, iron, phosphorus, etc. They are low in the Glycemic Index (GI) as such don't cause a
huge spike in blood sugar. Hence options 1 and 2 are correct.
• Millet consumption decreases triglycerides and C- reactive protein, thereby preventing cardiovascular
disease. All millets are rich in dietary fibre. Dietary fibre has water-absorbing and bulking property. It
increases transit time of food in the gut which helps in reducing the risk of inflammatory bowel
disease and acts as a detoxifying agent in the body.

Q 9.D
• There are unconsolidated sediments, deposited on the ocean floor. These are ocean deposits. They vary
from location to location. The ocean deposits can broadly be divided into two types - the terrigenous
deposits and the pelagic deposits.
• The terrigenous deposits are those which are found on the continental shelves and slopes and mainly
consist of the rock material derived because of wear and tear.
• The pelagic deposits are those which are found over deep-sea plains. Pelagic deposits are the most
conspicuous of all deposits covering about 75% of the total seafloor. This is because, except for fine
volcanic ash, little terrigenous material is carried into the deeps. The pelagic deposits consist of both
organic and inorganic material.
• Clays occur mainly as red clays in the deeper parts of the ocean basins and are particularly
abundant in the Pacific Ocean. Red clay is an accumulation of volcanic dust blown out from
volcanoes during volcanic eruptions.
• Poly Metallic Nodules are formed by the precipitation of metals from seawater over several million
years. Polymetallic nodules occur in most oceans of the world with the greatest abundance at the
vast abyssal floor at depths between 4000 m and 6000 m.
• The oozes are pelagic deposits because they are derived from the oceans. They are made of shelly and
skeletal remains of marine microorganisms with calcareous or siliceous parts. Oozes have a very fine,
flour-like texture and either occur as accumulated deposits or float about in suspension.
• Hence, both statements are not correct.

Q 10.C
• The Convention on International Trade in Endangered Species of Wild Fauna and
Flora (CITES) Monitoring the Illegal Killing of Elephants (MIKE) Programme is a site-based system
designed to monitor trends in the illegal killing of elephants, build management capacity and provide
information to help range States make appropriate management and enforcement decisions.
4 www.visionias.in ©Vision IAS
• MIKE aims to help range States improve their ability to monitor elephant populations, detect changes in
levels of illegal killing, and use this information to provide more effective law enforcement and strengthen
any regulatory measures required to support such enforcement.
• MIKE Sites in India:
o Chirang-Ripu Elephant Reserve: This elephant reserve is part of the Manas Biosphere Reserve and
made up of the Chirang and Ripu reserve forests. A large chunk of North-East India's wild Asian
Elephant population calls this area home.
o Deomali Elephant Reserve: It spreads over the Tirap and Changlang districts of Arunachal
Pradesh. Hence option 1 is correct.
o Dihing Patkai Elephant Reserve: Part of Dehing Patkai Wildlife Sanctuary is located in the
Dibrugarh and Tinsukia Districts of Assam and covers an area of 111.19 km (42.93 sq mi) rainforest.
It was declared a sanctuary on 13 June 2004. It is located in the Dehing patkai landscape which is a
dipterocarp-dominated lowland rainforest.
o Garo Hills Elephant Reserve: The Garo Hills are part of the Garo-Khasi range in Meghalaya, India.
They are inhabited mainly by tribal dwellers, the majority of whom are Garo people. It is one of the
wettest places in the world. The range is part of the Meghalaya subtropical forests eco-region.
o Eastern Dooars Elephant Reserve: The Dooars or Duars are the alluvial floodplains in eastern-
northeastern India that lie south of the outer foothills of the Himalayas and north of the Brahmaputra
River basin. This region is about 30 km (19 mi) wide and stretches over about 350 km (220 mi) from
the Teesta River in West Bengal to the Dhansiri River in Assam. The region forms the gateway to
Bhutan. It is part of the Terai-Duar savanna and grasslands ecoregion. A railway line runs through the
tea gardens and the various protected areas and a number of elephants have been killed in collisions
with trains
o Mayurbhanj Elephant Reserve: The Similipal-Kuldiha-Hadgarh Elephant Reserve popularly known
as Mayurbhanj Elephant Reserve in the State of Orissa. The area includes 3 protected areas i.e.
Similipal Tiger Reserve (2750.00 km²), Hadgarh Wildlife sanctuary (191.06 km²), and Kuldiha
wildlife sanctuary (272.75 km²).
o Shivalik Elephant Reserve: The Shivalik Elephant Reserve spans forest divisions in Dehradun,
Haridwar, Lansdowne, Haldwani, Tanakpur and Ramnagar. It also includes part of the Corbett Tiger
Reserve and the Rajaji National Park. Hence option 3 is correct.
o Mysore Elephant Reserve: is situated in Karnataka. It was established in the year 2002 and is spread
over an area of about 6,724 square kilometers. The Mysore-Ooty Highway passes through this
reserve.
o Nilgiri Elephant Reserve & Wayanad Elephant Reserve: The Theppakadu Elephant Camp forms
part of the Nilgiri Elephant Reserve. Banked by the Nilgiris, it has the Bandipur Tiger Reserve and the
Wynad Wildlife Sanctuary on two sides. The entire stretch is part of the Nilgiri Biosphere Reserve
and combined creates one of the largest uninterrupted forest habitats in the country. The area has
different kinds of vegetation in different regions and is the home of numerous animals. A place to
watch out for within these forests is the marshy 'Ombetta vayal' where elephants and bison flock.
• The Singhbhum Elephant Reserve (not a part of the MIKE site), the first Elephant Reserve of the
Country, was created in 2001 under the Project Elephant, comprising an area of 13,440 sq. Km. in East
and West Singhbhum and Saraikela-Kharsawan Districts (old Singhbhum District) for scientific and
planned management aimed at the conservation of Elephant habitats and viable population of wild Asiatic
Elephants in Jharkhand. Hence option 2 is not correct.

Q 11.C
• Green plants have the ability to make their own food. They do this through a process
called photosynthesis, which uses a green pigment called chlorophyll. Chlorophyll is located in a
plant’s chloroplasts, which are tiny structures in a plant’s cells. This is where photosynthesis takes place.
• Chlorophyll’s job in a plant is to absorb light - usually sunlight. Through photosynthesis, the plant uses
solar energy to convert carbon dioxide (absorbed from the air) and water into glucose, a type of sugar.
Plants use glucose together with nutrients taken from the soil to make new leaves and other plant parts.
The process of photosynthesis produces oxygen, which is released by the plant into the
air. Respiration occurs when glucose (sugar produced during photosynthesis) combines with oxygen to
produce useable cellular energy. This energy is used to fuel growth and all of the normal cellular
functions. Hence statement 1 is not correct.
• Chlorophyll gives plants their green color because it does not absorb the green wavelengths of white
light. That particular light wavelength is reflected from the plant, so it appears green. Hence statement 2
is not correct.
5 www.visionias.in ©Vision IAS
• Magnesium is an essential element throughout the whole growth period of a plant. Magnesium fulfills
several functions within the plant; it is a central component of chlorophyll which is supporting the
function to absorb sunlight during photosynthesis. Magnesium also acts as a phosphorus carrier in plants
and is essential for phosphate metabolism. Hence statement 3 is correct.

Q 12.D
• The olive ridley sea turtle, also known commonly as the Pacific ridley sea turtle, is a species of turtle in
the family Cheloniidae. The species is among the smallest of the world’s sea turtles and is found
primarily in the tropical regions of the Pacific, Indian, and Atlantic Oceans. Hence, statement 1 is
correct.
• The olive ridley is considered the most abundant sea turtle in the world, with an estimated 800,000 nesting
females every year. The olive ridley is one of two species of sea turtles that engage in "arribada" nesting,
where large groups of females gather offshore and come onto the beach to nest all at once. Nesting in
large groups may be a defense against predators or a result of environmental factors influencing nesting.
With many turtles coming ashore together and many nests subsequently hatching at the same time, it may
help to reduce predation.
• In the Indian Ocean, three arribada (mass nesting) beaches occur in Odisha, India-Gahirmatha, Devi
River mouth, and Rushikulya—with an estimated >100,000 nests per year. More recently, a new mass
nesting site was discovered in the Andaman Islands, India, with more than 5,000 nests reported in a
season. Hence, statement 2 is correct.
• The olive ridley is omnivorous, meaning it feeds on a wide variety of food items. Olive ridleys often
migrate great distances between feeding and breeding grounds.
• Operation Oliver is a program run by the Coast Guard to ensure safe breading for olive ridley sea
turtle in seawaters of Gahirmatha Marine Sanctuary in Kendrapara district of Odisha. Hence,
statement 3 is correct.

Q 13.C
• Animals living near colder climates are bigger in size because being heftier allows them several benefits
like reducing body heat loss. For most species of vertebrates, body mass increases the closer you get
to the poles.
• For many types of animals, it pays to be bigger in the colder climates that exist at high latitudes and
altitudes. Heftier animals have a smaller surface-area-to-volume ratio, which helps reduce heat
loss. A more massive organism has a smaller surface area-to-volume. This is known as Bergmann's
Rule.
• In the polar seas, aquatic mammals like seals have a thick layer of fat (blubber) below their skin that acts
as an insulator and reduces the loss of body heat.
• Many fish thrive in Antarctic waters where the temperature is always below zero. These animals protect
themselves from the excessive cold by developing cold hardiness. Cold hardiness is achieved by
developing extra solutes in the body fluid and special ice-nucleating proteins in the extracellular spaces.
The extra solutes which prevent cold are glycerol and anti-freezing proteins. They lower the freezing
points of body fluids.
• Hence, both statements are correct.

Q 14.C
• Statement 1 is correct: The Asian elephant is classified as Endangered by the International Union for the
Conservation of Nature (IUCN). Its population has declined by an estimated 50 percent over the past 75
years, and there are an estimated 20,000 to 40,000 Asian elephants left in the wild. India holds the largest
remaining population of wild Asian elephants. It accounts for nearly 60% of the Asian elephant
population in Southeast Asia and estimated at about 26,000 to 28,000.
• Statement 2 is not correct: Elephant corridors are narrow strips of land that connect two large habitats.
Elephant corridors in India are identified by states and notify them under either the Wild Life (Protection)
Act (WLPA), 1972, or the Environment Protection Act (EPA), 1986.
• Statement 3 is correct: India's Environment Ministry has declared the elephant a National Heritage
Animal in order to increase protective measures for the country's nearly 27,000 elephants.

Q 15.A
• Essential elements are those minerals whose absence in the plant is unable to complete a normal life cycle,
or that the element is part of some essential plant constituent or metabolite.
• These elements are further divided into two broad categories based on their quantitative requirements.
6 www.visionias.in ©Vision IAS
o Macronutrients are generally present in plant tissues in large amounts. The macronutrients
include carbon, hydrogen, oxygen, nitrogen, phosphorous, sulphur, potassium, calcium and
magnesium.
o Micronutrients or trace elements, are needed in very small amounts. These include iron,
manganese, copper, molybdenum, zinc, boron, chlorine and nickel. Hence statement 1 is not
correct.
• Role of Important Macronutrients:
o Nitrogen: Nitrogen is required by all parts of a plant, particularly the meristematic tissues and the
metabolically active cells. Nitrogen is one of the major constituents of proteins, nucleic acids,
vitamins and hormones.
o Phosphorus: Phosphorus is absorbed by the plants from the soil in the form of phosphate
ions. Phosphorus is a constituent of cell membranes, certain proteins, all nucleic acids and
nucleotides, and is required for all phosphorylation reactions. Hence statement 2 is correct.
o Potassium: In plants, this is required in more abundant quantities in the meristematic tissues, buds,
leaves and root tips. Potassium helps to maintain an anion-cation balance in cells and is involved in
protein synthesis, opening and closing of stomata, activation of enzymes and in the maintenance of
the turgidity of cells.
o Calcium: Calcium is required by meristematic and differentiating tissues. During cell division, it is
used in the synthesis of the cell wall, particularly as calcium pectate in the middle lamella. It is also
needed during the formation of the mitotic spindle. It accumulates in older leaves. Hence statement 3
is correct.
o Magnesium: It activates the enzymes of respiration, photosynthesis and is involved in the synthesis
of DNA and RNA. Magnesium is a constituent of the ring structure of chlorophyll and helps to
maintain the ribosome structure.
o Sulphur: Sulphur is present in two amino acids – cysteine and methionine and is the main constituent
of several coenzymes, vitamins (thiamine, biotin, Coenzyme A) and ferredoxin.

Q 16.C
• Gorumara National Park: It is a National Park in northern West Bengal. Located in the Dooars
region of the Himalayan foothills, it is a medium-sized park with grasslands and forests. It is primarily
known for its population of Indian rhinoceros.
o Gorumara is located in the Eastern Himalayas' submontane Terai belt. This region has rolling
forests and riverine grasslands and is known as the Dooars in West Bengal. The park is located on the
flood plains of the Murti River and Raidak River.
o The major river of the park is the Jaldhaka river, a tributary of the Brahmaputra river system. In this
regard, Gorumara is a significant watershed area between the Ganges and Brahmaputra river
systems. The park is very close to Jaldapara National Park and Chapramari Wildlife
Reserve. Hence option (c) is the correct answer.
• Buxa Tiger Reserve (BTR): It lies in the Alipurduar district of West Bengal. Its northern boundary runs
along the international border with Bhutan. The Sinchula hill range lies all along the northern side of
BTR and the eastern boundary touches that of the Assam state.
o National Highway No.31 C roughly runs along its southern boundary. It is the easternmost extension
of extreme bio-diverse North-East India and represents a highly endemic Indo-Malayan region.
o The fragile "Terai Eco-System" constitutes a part of this reserve.
o The Phibsoo Wildlife Sanctuary of Bhutan is contiguous to the north of BTR. Manas National
Park lies east of BTR. BTR, thus, serves as international corridor for Asian elephant
migration between India and Bhutan.
o To the south-west, the Chilapata Forests form an elephant corridor to the Jaldapara Wildlife
Sanctuary.
o The main trees are sal, Champa, gamhar, simul and chikrasi.
o Endangered species present in the reserve are leopard cat, Bengal florican, regal python, Chinese
pangolin, hispid hare, hog deer lesser adjutant , white-rumped vulture and slender-billed vulture.
• Jaldapara National Park: Jaldapara, the vast grassland with patches of riverine forests was declared a
sanctuary in 1941 for protection of the great variety of flora and fauna, particularly the one-horned
rhinoceros, an animal threatened with extinction.
o Drained by rivers Torsa, Malangi, Hollong, Chirakhawa, Kalijhora, Sissamara, Bhaluka and
Buri Torsa.

7 www.visionias.in ©Vision IAS


• Chapramari Wildlife Sanctuary:
o It is a wildlife sanctuary in the Dooars area of North Bengal. The forest is a continuation of the
Gorumara forests.
o The two forests are separated by the Murti river which passes through the otherwise continuous
forest. Whereas Gorumara is a National Park, Chapramari is only is a wildlife sanctuary only.
o Chapramari is famous for its elephant population, Gaur, Rhino, Leopard, etc.

Q 17.B
• Recent Context – Recently, India got elected as member of the UN Commission on the Status of Women
(CSW). India has been elected for a four-year term from 2021-2025.
• The UN Commission on the Status of Women (CSW) is a functional commission of the Economic and
Social Council (ECOSOC). It was established by ECOSOC in 1946. Hence statement 1 is not correct.
• It is the principal global intergovernmental body exclusively dedicated to the promotion of gender
equality and the empowerment of women. It is instrumental in promoting women’s rights, documenting
the reality of women’s lives throughout the world, and shaping global standards on gender equality and
the empowerment of women. Hence statement 2 is correct.
• In 1996, ECOSOC expanded the Commission’s mandate and decided that it should take a leading role in
monitoring and reviewing progress and problems in the implementation of the Beijing Declaration and
Platform for Action, and in mainstreaming a gender perspective in UN activities.
• 45 member states of the United Nations serve as members of the Commission at any one time. The
present elections were held for two seats in the Asia-Pacific States category with Afghanistan, India and
China in the fray. India and Afghanistan were elected whereas China was defeated.

Q 18.B
• The diversity of species (sometimes communities) in a landscape is generally observed in three different
scales - alpha diversity, beta diversity, and gamma diversity.
• Alpha diversity describes the species diversity within a community at a small scale or local scale,
generally the size of one ecosystem. When we casually speak of diversity in an area, more often than not
it refers to alpha diversity.
• Beta diversity describes the species diversity between two communities or ecosystems. It is at a larger
scale and looks to compare the species diversity between two separate entities that are often divided by a
clear geographical barrier like a river or a mountain ridge.
• Gamma diversity is studied at a very large scale—a biome—where species diversity is compared
between many ecosystems. It could range over areas like the entire slope of a mountain, or the entire
littoral zone of a sea-shore. Hence option (b) is the correct answer.

Q 19.A
• The Palamau Tiger Reserve was notified as one of India’s 1st nine Tiger Reserves under Project Tiger in
1973. It is located in the western part of the Chhotanagpur plateau with area expending to 1129.93 sq.km.
It is located in the districts of Latehar and Grahwa in the state of Jharkhand.
• The forest of Palamau is the catchment of the river North Koel.
• Palamau is the land of Palas and Mahua and represents the biological richness of the dry and moist
deciduous eastern peninsular forests with an interesting association of Sal and Bamboo. The verdant
landscape of the Reserve is clothed with stately Sal in the valleys and lower slopes, and with dry
deciduous forests on the upper slopes and on the mound caps in the northern part.
• Copious bamboo and scattered open grassy patches are common. Almost pure patches of Bel (Aegle
marmelos) are unique vegetation types mostly found in the northern part of the Reserve. There are many
waterfalls such as Baresand and Lodh Fall, which is the highest waterfall in the Jharkhand State. There is
a hot water spring -Tataha Pani, near Barwadih. Hence, option (a) is the correct answer.
• Some of the major problems associated with this Tiger reserve include:
o Severely affected by left-wing extremism
o Signs of biotic interference such as illegal felling of trees, overgrazing and infestation with weeds are
visible.
o Weeds such as Parthenium and Lantana camara are hindering the regeneration of native species.
• Kanha Tiger Reserve: It is located in Madhya Pradesh and is also known as Kanha–Kisli National Park.
Together with a surrounding buffer zone and the neighbouring Phen Sanctuary. The park hosts Bengal
tiger, Indian leopard, sloth bear, barasingha and dhole. It is also the first tiger reserve in India to officially
introduce a mascot, Bhoorsingh the Barasingha.

8 www.visionias.in ©Vision IAS


• Sanjay National Park is a national park in Madhya Pradesh. It is a part of the Sanjay-Dubri Tiger
Reserve. It is located in the Narmada Valley dry deciduous forests ecoregion.
• Bandhavgarh National Park is located in Madhya Pradesh. It was declared a national park in 1968 and
then became Tiger Reserve in 1993. The park has a large breeding population of leopards and various
species of deer.

Q 20.D
• Coral reefs have the highest biodiversity of any ecosystem on the planet—even more than a tropical
rainforest. Occupying less than one percent of the ocean floor, coral reefs are home to more than twenty-
five percent of marine life. The coral reefs are more diverse than tropical rainforests because coral reefs
have more than 1,000,000 species.
• They thrive in tropical oceans confined between 25 degrees North and 25 degrees south latitudes. Corals
are found mainly in the tropical oceans and seas because they require a high mean annual temperature
above 20 degrees celsius.
• The corals and algae have a mutualistic relationship. The coral provides the algae with a protected
environment and the compounds they need for photosynthesis. In return, the algae produce oxygen and
help the coral to remove wastes.
• Consequences of coral bleaching - The loss of coral reefs would be devastating to one of the world’s
largest pools of biodiversity. Coral reefs help seed the ocean and provide shelter and food to a complex
web of organisms that leads all the way up to the man.
o Coral reefs are recognized as net sinks for carbon. Coral bleaching and coral death will result in the
loss of carbon sinks. Hence option 1 is correct.
o Coral reefs act as natural barriers to shorelines, protecting them from the effects of the water. As the
coral reefs die, coastlines become more susceptible to damage and flooding from storms, hurricanes,
and cyclones. Hence option 3 is correct.
o New research reveals that global warming also affects fish who depend on corals. The Great Barrier
Reef is revered for its kaleidoscope of color. New international research reveals that coral bleaching
events not only whitewash corals but can also reduce the variety of fish occupying these highly valued
ecosystems. Hence option 2 is correct.

Q 21.C
• Wetlands have different characteristics. The most common feature of all wetlands is that the water table
(the groundwater level) is very near to the soil surface or shallow water covers the surface for at least part
of the year.
• The main characteristics of a wetland are determined by the combination of the salinity of the water in the
wetland, the soil type and the plants and animals living in the wetland.
• Dependent mostly on a combination of the above conditions, two major wetland types can be
distinguished– mineral and organic, such as marsh, swamp, fen and bog.

9 www.visionias.in ©Vision IAS


• Mineral Soil Wetlands:
o Marsh – a type of wetland ecosystem characterized by poorly drained mineral soils and by plant
life dominated by grasses. Marshes are common at the mouths of rivers, especially where extensive
deltas have been built. The marsh plants slow down the flow of water and allow for the nutrient
enriched sediments to be deposited, thus providing conditions for the further development of the
marsh. Hence, pair 2 is not correctly matched.
o Swamp – a wetland ecosystem characterized by mineral soils with poor drainage and by plant life
dominated by trees. Swamps are found throughout the world, most often in low-lying regions (with
poor drainage) next to rivers, which supply the swamp with water. Some swamps develop from
marshes that slowly fill in, allowing trees and woody shrubs to grow. Hence, pair 4 is not correctly
matched.
o The dominant vegetation, therefore, distinguishes the two major types of mineral soil wetlands:
grasses dominate marshes, while trees dominate swamps. Both marshes and swamps may be
freshwater or saltwater.
• Organic Soil Wetlands: Generally these wetlands are referred to as "peatlands" in recognition of their
common ability to form peat (organic soil produced by the accumulation of plant material). There are two
major types of peatlands – bogs and fens, both of which occur in similar climatic and geographic regions.
o Bog – a type of wetland ecosystem characterized by wet, spongy, poorly drained peaty soil,
dominated by the growth of bog mosses, Sphagnum, and heaths, particularly Chamaedaphne. Bogs
are usually acid areas, frequently surrounding a body of open water. Bogs receive water exclusively
from rainfall. Hence, pair 1 is correctly matched.
o Fen – a type of wetland ecosystem characterized by peaty soil, dominated by grasslike plants, grasses,
sedges, and reeds. Fens are alkaline rather than acid areas, receiving water mostly from surface
and groundwater sources. Hence, pair 3 is correctly matched.
o The two major types of organic soil wetlands are, therefore, distinguished by their hydrological
regime: bogs receive water mainly from precipitation, while fens are supplied with water mostly from
surface and groundwater sources.

Q 22.C
• Recent Context - Thailand has announced that it will scrap a Chinese-led-Kra canal project.
• Kra Canal Project - It refers to a proposed canal project that aims to connect the Gulf of Thailand with
the AndamanSea across the Kra Isthmus in southern Thailand. Hence option (c) is correct.
• China has proposed to build the Kra Canal with Thailand. It is also part of China’s Belt and Road
Initiative. It is being touted as a new gateway to China’s 21st Century Maritime Silk Road.
• Significane – The proposed Kra Canal, if constructed, will bypass the busiest Malacca Strait and will
connect the Gulf of Thai land (and the South China Sea) to the Andaman Sea. It will shorten China’s
access to the Indian Ocean.
• Thailand has dropped the project as it is turned out to be infeasible both economically and politically.
• Strait of Malacca - It is a narrow, 890 km (550 mi) stretch of water between the Malay Peninsula
(Peninsular Malaysia) and the Indonesian island of Sumatra. It serves as the main shipping channel
between the Indian Ocean and the Pacific Ocean and is one of the most important shipping lanes in the
world.

10 www.visionias.in ©Vision IAS


Q 23.A
• More than 99 percent of all organisms that have ever lived on Earth are extinct. As new species evolve to
fit ever-changing ecological niches, older species fade away. But the rate of extinction is far from
constant.
• At least a handful of times in the last 500 million years, 75 to more than 90 percent of all species on Earth
have disappeared in a geological blink of an eye in catastrophes we call mass extinctions. Though mass
extinctions are deadly events, they open up the planet for new forms of life to emerge.
• The reasons for mass extinctions are asteroid impacts, changes in Earth's Carbon Cycle due to large
igneous province eruptions, global warming, ocean acidification, and anoxia, a loss of dissolved oxygen in
the water.
• Some of the major mass extinctions are:
o Ordovician-Silurian extinction - 444 million years ago: massive glaciation locked up huge amounts
of water in an ice cap that covered parts of a large south polar landmass. As a result, sea levels
plummeted by hundreds of feet. Creatures living in shallow waters would have seen their habitats cool
and shrink dramatically, dealing a major blow.
o Late Devonian extinction - 383-359 million years ago: In several pulses across the Devonian, ocean
oxygen levels dropped precipitously, which dealt serious blows to conodonts and ancient shelled
relatives of squid and octopuses called goniatites.
o Permian-Triassic extinction - 252 million years ago: The extinction’s single biggest cause is the
Siberian Traps, an immense volcanic complex that erupted more than 720,000 cubic miles of lava
across what is now Siberia. The eruption triggered the release of at least 14.5 trillion tons of carbon,
more than 2.5 times what’d be unleashed if every last ounce of fossil fuel on Earth were dug up and
burned. Adding insult to injury, magma from the Siberian Traps infiltrated coal basins on its way
toward the surface, probably releasing even more greenhouse gases such as methane.
o Triassic-Jurassic extinction - 201 million years ago: At the end of the Triassic, Earth warmed an
average of between 5 and 11 degrees Fahrenheit, driven by a quadrupling of atmospheric CO2 levels.
This was probably triggered by huge amounts of greenhouse gases from the Central Atlantic
Magmatic Province, a large igneous province in central Pangaea, the supercontinent at the time.
Remnants of those ancient lava flows are now split across eastern South America, eastern North
America, and West Africa. The Central Atlantic Magmatic Province was enormous. Its lava volume
could cover the continental U.S. in a quarter-mile of rock.
o Cretaceous-Paleogene extinction - 66 million years ago: The Cretaceous-Paleogene extinction
event is the most recent mass extinction and the only one definitively connected to a major asteroid
impact. Some 76 percent of all species on the planet, including all nonavian dinosaurs, went extinct.
• Hence the correct answer is option (a).

Q 24.A
• Recent Context - Recently, India successfully tested an indigenously developed hypersonic technology
demonstration vehicle powered by a scramjet engine. The test was performed by the Defence Research
and Development Organisation (DRDO).
• About Hypersonic Technology Demonstrator Vehicle (HSTDV):
o HSTDV is an unmanned demonstration aircraft used for hypersonic flight test. It is developed
by Defence Research and Development Organisation (DRDO).
o It is not a weapon itself but is being developed as a carrier vehicle for hypersonic and long-range
cruise missiles
o The HSTDV cruise vehicle is mounted on a solid rocket motor, which takes it to a required altitude.
Once it attains certain mach numbers for speed, the cruise vehicle is ejected out of the launch vehicle.
After that, the scramjet engine is ignited automatically.
o The primary aim of the demonstration vehicle was to test the indigenously developed propulsion
system- airbreathing Scramjet engine. Hence statement 1 is correct.
▪ A scramjet engine is an improvement over the ramjet engine as it efficiently operates at
hypersonic speeds and allows supersonic combustion. Ramjets work most efficiently at
supersonic speeds around Mach 3. However, the ramjet efficiency starts to drop when the vehicle
reaches hypersonic speeds.
• What makes HSTDV a breakthrough?
o Hypersonic missiles are capable of maneuvering and flying faster than 5,000 kilometers per hour.
o The speed and maneuverability enable hypersonic missiles to penetrate most missile defenses, and
further compress the timelines for a response by a nation under attack.
11 www.visionias.in ©Vision IAS
o India has become the fourth country in the world to have hypersonic technology. Other countries are
the United States of America, Russia and China. Hence statement 2 is not correct.
o HSTDV is being developed as a carrier vehicle for hypersonic and long-range cruise missiles, and
will have multiple civilian applications including the launching of small satellites at low cost.

Q 25.C
• Rhododendron: Rhododendron is a large genus of flowering plants and is found mainly in Eastern
Himalayas, Western Himalayas and Nilgiris.
o Some plants of Rhododendron are evergreen and some are deciduous in nature.
o The species is found in varied habitats from subtropical forest to alpine shrubs, rhododendrons range
from dwarf shrubs to large trees.
o The cold, moist slopes and deep valleys of the eastern Himalayas form a conducive habitat for the
luxuriant growth of Rhododendron species and rich diversity in North Eastern States.
o Arunachal Pradesh is home to the highest number with 119 taxa (74 species, 21 sub species and 24
varieties) of the Rhododendron. The small State of Sikkim is home to 42 taxa (25 species, 11 sub-
species and six varieties) while 10 taxa are found in Manipur, four in Mizoram and 11 in Nagaland.
• Kanchendzonga Biosphere Reserve: It was inscribed to the UNESCO World Heritage Sites list in July
2016, becoming the first "Mixed Heritage" site of India. It was recently included in the UNESCO Man
and the Biosphere Programme. World’s third highest peak, Mt. Khangchendzonga, is situated
within the Park. Numerous lakes and glaciers, including the 26 km long Zemu Glacier are a part of the
Biosphere reserve.
• The epiphytes and lianas are abundant here. Besides, there are about 30 species of rhododendrons
recorded and out of over 42 confirmed mammal species belonging to 16 families in the area.
• Singalila National Park : Rhododendrons are regarded by many as the best flowering evergreen plants in
the temperate landscape. Singalila National Park has 18 recorded species of rhododendrons.
• Neora Valley National Park: It is contiguous with Sikkim and Bhutan at its northern and north-eastern
boundaries respectively and links the Pangolakha Wildlife Sanctuary in Sikkim and the Toorsa Strict
Reserve of Bhutan. It is also an integral part of the Kanchenjunga Landscape.
• The southern boundaries of the Park are adjoining to the forests of Jalpaiguri district which have
connectivity with the Chapramari Wildlife Sanctuary and the Gorumara National Park. The highest
point is Rachela Danda. It is the land of the elegant red panda in the pristine undisturbed natural habitat
.The common species of Rhododendrons are Rhododendron arboruem, R. barbatum, R.falconeri, R.
dalhousiae.
• Sajnakhali Wildlife Sanctuary is a 362 km2 area in the northern part of the Sundarbans delta in
South 24 Parganas district, West Bengal, India. The area is mainly mangrove scrub, forest and
swamp. It is home to a rich population of different species of wildlife, such as water fowl, heron, pelican,
spotted deer, rhesus macaques, wild boar, tigers, water monitor lizards, fishing cats, otters, Olive ridley
turtle, crocodiles, Batagur terrapins, and migratory birds. Hence option c is correct.

Q 26.C
• Hydrothermal vents occur in volcanically active areas of the seafloor like mid-ocean ridges, where
tectonic plates are pushing and pulling above magma hotspots in Earth’s crust. Hot springs on the ocean
floor are called hydrothermal vents. The heat source for these springs is the magma (molten rock) beneath
submarine volcanoes.
• In these areas, super-heated gases and chemically rich water erupt from the ground at
temperatures of up to 400°C.The deep-ocean temperatures could change so drastically—from near
freezing to 400 °C. The hot hydrothermal fluids mix with near-freezing seawater to form very fine-
grained sulfide minerals. Hot seawater in hydrothermal vents does not boil because of the extreme
pressure at the depths where the vents are formed.
• The vent organisms can survive and thrive in extreme pressures and temperatures and in the presence of
toxic mineral plumes. The conversion of mineral-rich hydrothermal fluid into energy is a key aspect of
these unique ecosystems. Through the process of chemosynthesis, bacteria provide energy and nutrients to
vent species without the need for sunlight.
• At deep hydrothermal vents, specialized bacteria can convert the sulfur compounds and heat into
food and energy. As these bacteria multiply, they form thick mats on which animals can graze. The
bacteria form symbiotic relationships with animals, (e.g., giant tube worms) and live in the animals’
tissues, creating energy in return for receiving protection from predators. This is chemosynthetic
symbioses, in which both organisms involved are believed to benefit from the relationship. These
specialized bacteria form the bottom of the deep hydrothermal vent food web, and many animals rely on
12 www.visionias.in ©Vision IAS
their presence for survival, including deep-sea mussels, giant tube worms, yeti crabs, and many other
invertebrates and fishes.
• Hence, both statements are correct.

Q 27.B
• Conservation reserves and community reserves in India are terms denoting protected areas of India which
typically act as buffer zones to or connectors and migration corridors between established national
parks, wildlife sanctuaries, and reserved and protected forests of India.
• Such areas are designated as conservation areas if they are uninhabited and completely owned by the
Government of India but used for subsistence by communities and community areas if part of the lands is
privately owned.
• These protected area categories were first introduced in the Wildlife (Protection) Amendment Act of 2002
− the amendment to the Wildlife Protection Act of 1972. Hence statement 1 is not correct.
• Gogabeel is an ox-bow lake in Bihar’s Katihar district.It has been declared as the state’s first
‘Community Reserve’.Hence statement 2 is correct.
• The water body was also been notified as both Community Reserve and a conservation reserve.
• Gogabeel is formed from the flow of the rivers Mahananda and Kankhar in the north and the Ganga
in the south and east. It is the fifteenth Protected Area(PA) in Bihar. Hence statement 3 is correct.
• Gogabeel is a permanent waterbody although it shrinks to some extent in the summer but never dries
completely. More than 90 bird species have been recorded from this site of which about 30 are migratory.
• An oxbow lake is a crescent-shaped lake lying alongside a winding river. The oxbow lake is created over
time as erosion and deposits of soil changed the river’s course.

Q 28.A
• Coral reefs are underwater structures built by tiny sea animals. Hard corals, which have a stone-like
skeleton, grow into reefs on the edges of islands and continents. Depending on the depth, they can be
shallow-water corals or deep-sea corals.
• Deep-sea corals are defined as corals that live at depths greater than 50 meters, but most species live in
depths several hundred meters deep, in cold, dark, rocky habitats, often far from shore.
• Unlike shallow-water coral reefs, which are limited to warm tropical waters, deep-sea corals are found
throughout the world’s oceans, from tropical to polar regions. Like their shallow counterparts, deep-sea
reefs are created by stony corals that form large geological structures over thousands of years. Hence,
statement 1 is correct.
• While the diversity of corals that build reefs in shallow waters is very high, and includes hundreds of
species, there are only six species of deep-sea stony corals that create reefs. The most common of these is
Lophelia pertusa, which forms massive reefs throughout the Atlantic Ocean, including the Gulf of Mexico
and the South Atlantic Bight. Hence, statement 2 is not correct.
• Shallow-water corals rely on photosynthetic algae that live within the corals and provide their hosts with
nutrition. However, algae cannot survive where there is no light, so deep-sea corals need to rely on
alternate sources of food to obtain their nutrition, either by feeding on organic material falling from the
surface or by feeding on small plankton. Hence, statement 3 is correct.
• Because deep-sea corals lack algae, they are bright white in coloration, rather than the brown and green
colors than we see in shallow reefs. Other types of corals such as gorgonians, black corals, and soft corals,
add diversity and color to deep-sea coral ecosystems. Hence, statement 4 is not correct.
• As with shallow reefs, the structure created by deep-sea corals provides habitat for diverse communities of
other invertebrates and fishes, some of which are economically valuable.

Q 29.A
• An ecological pyramid is a graphical representation designed to show the biomass or
bioproductivity at each trophic level in a given ecosystem.
• Ecological pyramids express the food or energy relationship between organisms at different trophic levels.
This, relationship is expressed in terms of number, biomass or energy.
• The base of each pyramid represents the producers or the first trophic level while the apex represents
tertiary or top level consumer. The three types of ecological pyramids that are usually studied are (a)
pyramid of number; (b) pyramid of biomass and (c) pyramid of energy.
o A pyramid of numbers shows graphically the population, or abundance, in terms of the number of
individual organisms involved at each level in a food chain.
o A pyramid of biomass shows the relationship between biomass and trophic level by quantifying the
biomass present at each trophic level of an ecological community at a particular time. It is a graphical
13 www.visionias.in ©Vision IAS
representation of biomass (total amount of living or organic matter in an ecosystem) present in unit
area in different trophic levels.
o A pyramid of energy or pyramid of productivity shows the production or turnover (the rate at which
energy or mass is transferred from one trophic level to the next) of biomass at each trophic
level. Pyramid of energy is always upright, can never be inverted.
• However, there are certain limitations of ecological pyramids such as it does not take into account the
same species belonging to two or more trophic levels. It assumes a simple food chain, something that
almost never exists in nature; it does not accommodate a food web. Moreover, saprophytes are not
given any place in ecological pyramids even though they play a vital role in the ecosystem. Hence
the correct option is (a)

Q 30.B
• Keeping in view the recent human encroachment, the Indian Government did take effective initiatives to
conserve wildlife in the country, and amongst it, the most commendable initiative is the Wildlife
Protection Act of 1972, which prohibits the trade of rare and endangered species.
• The Government has also undertaken initiatives like:
o Project Tiger: One of the most successful wildlife conservation ventures 'Project Tiger' which
was initiated way back in 1972, has not only contributed to the conservation of tigers but also of the
entire ecosystem. This project is sponsored by the Ministry of Environment Forest and Climate
Change. About 50 tiger reserves situated in more than 17 regions are part of this project which
conducts assessments of the number of tigers, their habitat, hunting habits under the supervision of the
Tiger Task Force. Project Tiger has seen significant success in the recovery of the habitat and
increases in the population of the tigers in the reserve areas, from a scanty 268 in 9 reserves in 1972 to
above 1000 in 28 reserves in 2006 to 2967 tigers in 2020.
o Project Crocodile: Started in 1975, this project is yet another successful venture by the Government
of India to conserve the Indian Crocodiles, whose species were on the verge of extinction once. The
main objectives of the crocodile project is to protect the remaining population of crocodiles and their
natural habitat by establishing sanctuaries; to promote captive breeding; to improve management, and
to involve the local people in the project intimately. It is worth noticing that with the initiation of the
Crocodile Conservation Project, 4000 gharials/Aligator, 1800 mugger/crocodile and 1500 saltwater
crocodiles could be restocked.
o Project Elephant: Initiated in 1992 by the Government of India Project Elephant aims at conserving
elephants and their habitat and of migratory routes by developing scientific and planned management
measures. Under the project welfare of the domestic elephants is also considered, issues like
mitigation of human-elephant conflict are also taken care of. The project’s endeavour is to strengthen
the measures for the protection of elephants against poachers and unnatural death. Other initiatives
include the MIKE programme, 'Hathi Mere Sathi' initiative.
o Project Snow-Leopard: With a goal to safeguard and conserve India’s unique natural heritage of
high altitude wildlife populations and their habitats by promoting conservation through participatory
policies and actions this project was initiated by the Government in 2009 across all biologically
important landscapes in the Himalayan high altitudes in the states of Jammu & Kashmir,
Himachal Pradesh, Uttarakhand, Sikkim, and Arunachal Pradesh. Hence the correct answer is
option (b).
Q 31.D
• Mangroves are the characteristic littoral plant formation of tropical and subtropical sheltered
coastlines. Mangroves are trees and bushes growing below the high water level of spring tides which
exhibits a remarkable capacity for saltwater tolerance.
• Their physiological adaptation to salinity stress and to waterlogged anaerobic mud is high. With their
roots submerged in water, mangrove trees thrive in hot, muddy, salty conditions that would quickly kill
most plants. It is done through a series of adaptations.
o It produces pneumatophores (blind roots) to overcome the respiration problem in the anaerobic soil
conditions.
o Leaves are thick and contain salt secreting glands.
o Some mangroves have adaptations to conserve water. These include a thick waxy cuticle (skin on
the leaf) or dense hairs to reduce transpiration.
o Mangroves exhibit Viviparity mode of reproduction. i.e. seeds germinate in the tree itself (before
falling to the ground). This is an adaptative mechanism to overcome the problem of germination in
saline water. Hence option (d) is the correct answer.
• In India, mangroves are found in all coastal states and Andaman and Nicobar islands.
14 www.visionias.in ©Vision IAS
Q 32.D
• When alien species are introduced unintentionally or deliberately for whatever purpose, some of
them turn invasive and cause the decline or extinction of indigenous species.
• The Nile perch introduced into Lake Victoria in east Africa led eventually to the extinction of an
ecologically unique assemblage of more than 200 species of cichlid fish in the lake. The environmental
damage caused and the threat posed to our native species by invasive weed species like carrot
grass (Parthenium), Lantana and water hyacinth (Eichhornia), Hemp/Marijuna. Hence the correct
option is (d)
• The recent illegal introduction of the African catfish Clarias gariepinus for aquaculture purposes is posing
a threat to the indigenous catfishes in our rivers.
• Centre for Biodiversity Policy and Law (CEBPOL) and the National Biodiversity Authority of India
has developed a comprehensive methodology for the identification of invasive species of plants and
animals.

Q 33.C
• The Irrawaddy dolphin is a critically endangered species. It is found near sea coasts and in estuaries
and rivers in parts of the Bay of Bengal and Southeast Asia. In India, it is found in Lake Chilika.
Hence, pair 1 is not correctly matched.
• National Chambal Sanctuary, also called the National Chambal Gharial Wildlife Sanctuary, is a 5,400
km2 (2,100 sq mi) tri-state protected area in northern India for the protection of the Critically Endangered
gharial. More than 5,000 gharials (Gavialis gangeticus) were born in the latest hatching season (June-July,
2019) at the National Chambal Sanctuary. Hence, pair 2 is correctly matched.
• The hump-backed mahseer is a large freshwater fish which is also called the tiger of the river. It is
found only in the Cauvery river basin (including Kerala’s Pambar, Kabini and Bhavani rivers). Recently
it has been added to the IUCN's Red List as Critically Endangered. The inclusion of the mahseer in the
Red List, an inventory of the conservation status of the world’s species, was possible only once the fish
got its scientific name last June—Tor remadevii. Shoal (an international organisation working to conserve
freshwater species) has initiated ‘Project Mahseer’ in India, along with other stakeholders to enable
conservation action for the hump-backed mahseer. Shoal is a new partnership aimed at engaging a wide
range of organisations to accelerate and escalate action to save the most threatened fish and other
freshwater species. Hence, pair 3 is correctly matched.

Q 34.D
• Nokrek Biosphere Reserve: The Nokrek Biosphere Reserve is located in the northeast of India on the
Tura Range, which forms part of the Meghalaya Plateau (average altitude: 600 metres). The entire area is
mountainous and Nokrek is the highest peak of the Garo hills, rising up 1,412 metres. Hence statement 1
is correct.
o The park covers an area of about 47 sq km and has been listed under UNESCO’s Man and
Biosphere Reserve program. Hence statement 2 is correct.
o The biosphere reserve contains major rivers and streams that form a perennial catchment system.
Examples include the Ganol, Dareng and Simsang rivers, of which the latter is the longest and
largest. The Simsang originates in the north of the Biosphere Reserve, the Dareng from the southern
peaks, and the Ganol flows westward into the Brahmaputra River, which supplies water to numerous
towns. Hence statement 3 is not correct.
o Nokrek has a huge population of rare plants and animals. Some of them include a very rare species of
citrus-indica or wild orange, found at this place, which the locals call memang narang. Literally
translated, it means the orange of the spirits. The Indian wild orange (citrus indica) which is believed
to be progenitor of all citrus species in the world. Hence statement 4 is correct.
o Abundant wildlife including herds of wild elephants, animal species like leopard, pangolin, hoolock
gibbon, python, hornbill, besides rare orchids abound in the sanctuary. The last few red pandas of the
world also call this place home. One can also spot bird species like hornbill, peacock and pheasant.
o Other attractions are the Simsang River Game Reserve, Ronbang Dare Waterfalls and the Nokrek
Peak.

Q 35.D
• Water often contains organic materials that are decomposed by microorganisms, which use oxygen in the
process. The amount of oxygen consumed by these organisms in breaking down the waste is known as the
biochemical oxygen demand or BOD. Other sources of oxygen-consuming waste include stormwater
runoff from farmland or urban streets, feedlots, and failing septic systems.
15 www.visionias.in ©Vision IAS
• Oxygen is measured in its dissolved form as dissolved oxygen (DO). If more oxygen is consumed than is
produced, dissolved oxygen levels decline and some sensitive animals may move away, weaken, or die.
• DO levels fluctuate seasonally and over a 24-hour period. They vary with water temperature and
altitude. Cold water holds more oxygen than warm water and water holds less oxygen at higher
altitudes. Hence in the question statement, nothing can be conclusively said.
• Thermal discharges, such as water used to cool machinery in a manufacturing plant or a power plant, raise
the temperature of water and lower its oxygen content. Aquatic animals are most vulnerable to lowered
DO levels in the early morning on hot summer days when stream flows are low, water temperatures are
high, and aquatic plants have not been producing oxygen since sunset.
• In contrast to lakes, where DO levels are most likely to vary vertically in the water column, the DO in
rivers and streams changes more horizontally along the course of the waterway. This is especially true in
smaller, shallower streams. In larger, deeper rivers, some vertical stratification of dissolved oxygen might
occur. The DO levels in and below riffle areas, waterfalls, or dam spillways are typically higher than those
in pools and slower-moving stretches."
Q 36.A
• In the deep ocean, carbonate particles in bottom sediments are supplied by the die-off of surface plankton
that secretes calcium carbonate tests or shells. When these shells fall below a certain water depth, they
begin to dissolve as ocean waters become undersaturated with respect to calcium carbonate because of
increasing pressure, decreasing temperature and increasing amounts of dissolved CO2. With increasing
depth, the rate of dissolution increases.
• The carbonate compensation depth, or CCD, is defined as the water depth at which the rate of
supply of calcium carbonate from the surface is equal to the rate of dissolution. As long as the ocean
floor lies above the CCD, carbonate particles will accumulate in bottom sediments, but below, there is no
net accumulation. The seafloor near ocean ridges is typically above the CCD and carbonates are important
sediment constituents, but with spreading and cooling, the seafloor descends below the CCD and deep-sea
clays become predominant. Hence, statement 1 is correct.
• As the pH of the ocean falls, it results in a shallowing of the lysocline and the CCD, thus exposing more
of the shells trapped in the sediments to undersaturated conditions causing them to dissolve. Thus ocean
acidification causes this horizon to rise vertically in the water column so more and more calcifying
organisms will be exposed to undersaturated water and thus vulnerable to the dissolution of their
shells and skeletons. Hence, statement 2 is not correct.
Q 37.C
• Recently, Qatar has brought about a change in its labor laws, scrapping rules requiring migrant
workers to take their employers’ permission before changing jobs, and setting the monthly
minimum wage at about $274, an increase of over 25 percent. The reforms, which were announced by
the Emir of Qatar in October 2019, were signed into law.
o The first reform has abolished the unjustified ‘kafala system’ or requirement for a “no
objection certificate” that migrant workers needed to get from their employers before changing
jobs. Hence option (b) is the correct answer.
o The second reform involves increasing the minimum wage by 25 percent to $274 or 1000 Qatari
riyals. These reforms are now applicable to workers of all nationalities and in all sectors, including
domestic workers who were previously excluded.
Q 38.B
• Recent Context - Ministry of Housing & Urban Affairs has launched the Climate Smart Cities
Assessment Framework (CSCAF) 2.0
• About CSCAF 2.0 - Climate Centre for Cities (C-Cube) at the National Institute of Urban Affairs under
the aegis of The Ministry of Housing and Urban Affairs has launched the Climate Smart Cities
Assessment Framework (CSCAF) 2.0. Hence statement 1 is not correct.
• It is a first of its kind city assessment framework on climate relevant parameters. The framework serves as
a tool for cities to assess their present climate situation and provides a roadmap for cities to adopt and
implement climate actions to help mainstream resilience.
• In the last decade, an increasing frequency of cyclones, floods, heat waves, water scarcity and drought-
like conditions have had adverse impacts on many of our cities. Such extreme events and risks cause loss
of life as well as impact the economic growth. In this context, CSCAF initiative intends to inculcate a
climate-sensitive approach to urban planning and development in India. Hence, statement 2 is correct.
• The framework has 28 indicators across five categories namely;
o Energy and Green Buildings,
o Urban Planning, Green Cover & Biodiversity,
o Mobility and Air Quality,
16 www.visionias.in ©Vision IAS
o Water Management and
o Waste Management.
Q 39.C
• Carnivorous plant, sometimes called insectivorous plants are adapted for capturing and digesting insects
and other animals by means of ingenious pitfalls and traps.
• The insectivorous plants often have several attractions such as brilliant colours, sweet secretions and
other curios to lure their innocent victims.
• Insectivorous plants mainly grow in humid wildernesses and bogs where plenty of sunlight and
moisture can be found. They are usually associated with rain-washed, nutrient-poor soils, or wet and
acidic areas that are ill drained. Hence statement 2 is correct.
• These plants (e.g. pitcher plant) are green and carry out photosynthesis to obtain a part of the food
required by them. They supplement their photosynthetic food production by trapping insects and
digesting their nitrogen-rich bodies. Hence statement 1 is not correct.
• Some of the insectivorous plants found in India are Drosera or Sundew, Aldrovanda, Nepenthes,
Utricularia or Bladderworts, Pinguicula or Butterwort. Hence statement 3 is correct.
• Hence option (c) is the correct answer.
Q 40.D
• Dead zones are low-oxygen or hypoxic, areas in the world’s oceans and lakes. The Earth currently has
more than 400 oceanic dead zones, with the count doubling every decade.
• Natural causes include coastal upwelling and changes in wind. Natural dead zones can be found
worldwide, particularly near regions where strong upwelling occurs. These natural dead zones have
typically had low oxygen levels over huge lengths of time. During periods of prolonged upwelling, each
successive wave of upwelling fertilizes more phytoplankton blooms. As these blooms decay, the
continental shelf's low-oxygen waters expand, lose more oxygen and move closer to shore.
• Stratification of the water column due to density differences also contributes to the formation of
hypoxic or "dead zones". The cooler, saltier (denser) water from the ocean, flows underneath the
warmer, fresher (less dense) water from the rivers. A boundary, called a pycnocline, forms and oxygen
consumed below the pycnocline cannot be replenished from above.
• Dead zones occur because of a process called eutrophication, which happens when a body of water
gets too many nutrients, such as phosphorus and nitrogen. With too many nutrients, cyanobacteria grow
out of control, which can be harmful. Human activities are the main cause of these excess nutrients being
washed into the ocean. For this reason, dead zones are often located near inhabited coastlines.
• Hence, option (d) is correct.
Q 41.B
• Earth’s biodiversity based on the currently available species inventories includes, more than 70 per cent
of all the species recorded are animals (Vertebrates - Fishes, mammals, birds, reptiles, amphibians;
Invertebrates - Insects, molluscs, crustaceans), while plants (including algae, fungi, bryophytes
(Mosses), Lichens, Pteridophytes (Ferns and allies), gymnosperms and angiosperms) comprise no
more than 22 per cent of the total. Hence the correct answer is option (b).
• Among animals, insects are the most species-rich taxonomic group, making up more than 70 per cent of
the total. That means, out of every 10 animals on this planet, 7 are insects.
• Although India has only 2.4 per cent of the world’s land area, its share of the global species diversity
is an impressive 8.1 per cent. That is what makes our country one of the 12 mega diversity countries of
the world. Nearly 45,000 species of plants and twice as many animals have been recorded from India.
• Monera is a kingdom that contains unicellular organisms with a prokaryotic cell organization (having no
nuclear membrane), such as bacteria. They are single-celled organisms with no true nuclear membrane
(prokaryotic organisms).
• The Protista, or Protoctista, are a kingdom of simple eukaryotic organisms, usually composed of a single
cell or a colony of similar cells. Protists live in water, in moist terrestrial habitats, and as parasites and
other symbionts in the bodies of multicellular eukaryotes.
• Fungi are a kingdom of usually multicellular eukaryotic organisms that are heterotrophs (cannot make
their own food) and have important roles in nutrient cycling in an ecosystem. Fungi reproduce both
sexually and asexually, and they also have symbiotic associations with plants and bacteria.
Q 42.A
• Recent context – Palestine has quit its current chairmanship of Arab League meetings, in protest over
Israel deals with the United Arab Emirates and Bahrain
• Arab League - The League of Arab States, commonly known as the Arab League, is an
intergovernmental organization of countries whose peoples are mainly Arabic speaking or where Arabic is
an official language.
17 www.visionias.in ©Vision IAS
• It was formed in 1945 when Arab League charter was adopted by seven newly independent Arab States –
Egypt, Iraq, Jordan, Lebanon, Saudi Arabia, Syria and North Yemen.
• Its head quarter is located at Cairo, Egypt.
• Its purpose is to promote closer political, economic, cultural and social relations among the
members. Hence statement 2 is not correct.
• Observer countries - Brazil, Eritrea, India and Venezuela. Hence statement 1 is correct.
• A council composed of representatives from the member states works together to settle disputes
peacefully. The league has five major committees: political, economic, social and cultural, legal and
Palestinian affairs. Each member has one vote on the council. Decisions are only binding to the states that
have voted for them. There is no mechanism to compel members’ compliance with its resolutions.
Q 43.D
• Ocean acidification refers to a reduction in the pH of the ocean over an extended period of time, caused
primarily by uptake of carbon dioxide (CO2) from the atmosphere. For more than 200 years, or since
the industrial revolution, the concentration of carbon dioxide (CO2) in the atmosphere has increased due
to the burning of fossil fuels and land-use change. The ocean absorbs about 30 per cent of the CO2 that is
released in the atmosphere, and as levels of atmospheric CO2 increase, so do the levels in the ocean.
• Some parts of the ocean are naturally acidic, such as at underwater hydrothermal vents and cold seeps.
These openings occur on the seafloor and are caused by underground volcanic activity.
• Various factors can locally influence the chemical reactions of CO2 with seawater and add to the effects
of ocean acidification. For example:
o Acid rain: Acid rain can have a pH between 1 and 6 and has an impact on surface ocean chemistry. It
has a major effect on ocean acidification locally and regionally but very small globally.
o Eutrophication: Coastal waters are also affected by excess nutrient inputs, mostly nitrogen, from
agriculture, fertilizers and sewage. The resulting eutrophication leads to large plankton blooms, and
when these blooms collapse and sink to the sea bed the subsequent respiration of bacteria
decomposing the algae leads to a decrease in seawater oxygen and an increase in CO2 (a decline in
pH).
• Hence, option (d) is correct.

Q 44.B
• Germany, current EU president and Europe's biggest economy, has launched its Indo-Pacific
strategy with India that is expected to play a key role in Berlin’s outreach in the region where
China’s aggressive foreign policy has rattled countries.
• The strategy released has several indirect references to Chinese behaviour that challenged rules-based
world order. Berlin’s strategy also suggested opening a dialogue with institutions where India plays a key
role -- Bay of Bengal Initiative for Multi-Sectoral Technical and Economic Cooperation and Indian Ocean
Rim Association -- in the areas of business and maritime safety, as well as disaster risk management.
• The South China Sea disputes involve both island and maritime claims among several sovereign states
within the region, namely Brunei, the People's Republic of China (PRC), the Republic of China
(ROC/Taiwan), Indonesia, Malaysia, the Philippines, and Vietnam.
• The disputes include the islands, reefs, banks, and other features of the South China Sea, including
the Spratly Islands, Paracel Islands, Scarborough Shoal, and various boundaries in the Gulf of
Tonkin. There are further disputes, including the waters near the Indonesian Natuna Islands, which many
do not regard as part of the South China Sea.
• The Senkaku Islands dispute, or Diaoyu Islands dispute, concerns a territorial dispute over a group of
uninhabited islands known as the Senkaku Islands in Japan, the Diaoyu Islands in the People's Republic
of China (PRC), and Tiaoyutai Islands in the Republic of China (ROC or Taiwan).
• The Falkland Islands (Islas Malvinas) are a remote South Atlantic archipelago. Sovereignty over the
Falkland Islands is disputed by Argentina and the United Kingdom. Hence option (b) is the correct
answer.
Q 45.B
• Lake is the largest freshwater lake in North East India and is famous for the phumdis (heterogeneous mass
of vegetation, soil and organic matter at various stages of decomposition) floating over it.
• A large, but the shrinking freshwater lake and associated swamplands supplied by several streams. Thick,
floating mats of weeds covered with soil are a characteristic feature of this lake.
• The lake is used extensively by local people as a source of water for irrigation and domestic use and is an
important wintering and staging area for waterbirds, particularly ducks. It also plays an important role in
flood control.
18 www.visionias.in ©Vision IAS
• Included on the Montreux Record in 1993 as a result of ecological problems such as deforestation in the
catchment area, the infestation of water hyacinth, and pollution. The construction of a dam for
hydroelectric power generation and irrigation purposes has caused the local extinction of several native
fish species.
• This ancient lake plays an important role in the economy of Manipur. It serves as a source of water for
hydropower generation, irrigation and drinking water supply. The lake is also a source of livelihood for
the rural fishermen who live in the surrounding areas and on phumdis, also known as "phumshongs".
• Hence, option (b) is correct.
• Rudrasagar Lake is located in Tripura. It is identified as one of the wetlands of National Importance as
well as a wetland of international importance.
• Chilika Lake is a brackish water lagoon in Odisha at the mouth of the Daya River. It has been listed as a
tentative UNESCO World Heritage site. It was designated the first Indian wetland of international
importance under the Ramsar Convention.
• Kolleru Lake a freshwater lake in Andhra Pradesh located between Krishna and Godavari deltas. It was
declared as a wildlife sanctuary and designated a wetland of international importance under Ramsar
Convention.
Q 46.C
• Plants grow in diverse environments including desserts. They show different adaptations to survive in
those environments. Based on the environment they grow, plants can be categorized in the following
categories.
• Hydrophytes: These are the plants growing in aquatic environments. Hydrophytic plants can be seen in
freshwater bodies as well as in marine environments. Hydrophytes show different adaptations to survive
in water. These adaptations include thin cuticle or no cuticle, presence of a high number of stomata,
stomata kept open all the time, leaves possessing air sacs, fewer roots density or no root system etc.
• Mesophytes: Mesophytes are plants that grow in typically average conditions. They are adapted to
an adequate or average supply of water. And also they are able to live in average temperature
conditions. Mesophytes contain a well-developed root system. They possess large leaves and average
lengthier cuticle.
• Xerophytes: Xerophytes are the plants that live in dry habitats. They are adapted to survive in an
extremely limited water supply. Xerophytes can be seen in deserts. Their adaptations include thick cuticle,
small leaves with reduced leaf lamina, low stomata density, sunken stomata, stomatal hairs, rolled leaves,
extensive roots, etc.
• Hygrophytes: The plants of this group are invariably moisture and shade-loving. Their root system
and vascular tissue are poorly developed. Their stem and roots are soft and spongy and show stunted
growth. The stem is mostly modified as underground rhizome which is also an organ for propagation.
• Halophytes: The plants that inhabit saline soil with high concentrations of salts or saline water belong to
this ecological group. These plants have succulent leaves and sometimes the stem is also succulent. In
certain cases, leaves are modified into spines. Such xerophytic adaptations are the result of the reaction of
an excess of salts on the plants. Hence pair 3 is not correctly matched.
• Phreatophyte: A phreatophyte is a deep-rooted plant that obtains a significant portion of the water that
it needs from the phreatic zone (zone of saturation) or the capillary fringe above the phreatic zone. They
can usually be found along streams where there is a steady flow of surface or groundwater in areas where
the water table is near the surface. Hence pair 2 is correctly matched.
• Tropophyte: These plants are adapted to climatic conditions in which periods of heavy rainfall
alternate with periods of drought. In the Northern deciduous forests, typical tropophytes adapt to a
mesophytic summer and a xerophytic winter. Hence pair 1 is not correctly matched.
Q 47.D
• Hydrophytes are those plants, which grow in aquatic habitats like lakes, ponds, rivers, seas. Structural
modifications in Hydrophytes include:
o The free-floating species usually grow on the surface of the water. The root system is greatly reduced
since the entire plant remains in intimate contact with water and absorption occurs through the
surface of the plant body.
o The leaves have stomata only on the surface not touching the water.
o Leaves are most prominent and well adapted to carry out absorption, photosynthesis, and rapid
vegetative multiplication and provide buoyancy due to large air spaces in them.
o In submerged water plants like Hydrilla, the adaptations have developed in response to the high
density of the surrounding aqueous medium and low light intensity. The stem organs develop large air

19 www.visionias.in ©Vision IAS


cavities or hollow spaces called lacunae. The air-filled lacunae in stem tissues provide sufficient
buoyancy to pull up the shoot for maintaining the plant in a well-expanded shape in the water.
o The floating leaves in most species are quite similar in shape and features of adaptation. The leaves
are peltate, usually circular in shape and have a strong leathery texture. The upper leaf surface is
waxy and smooth, as a result, water glides off and does not adhere.
o Mechanical tissues such as sclerenchyma, woody xylem, and secondary wood, bark are normally
present in the terrestrial plants to provide mechanical support but are absent in hydrophytes.
o Therefore, the presence of lacunae tissue, filled with air and the absence of mechanical tissues are two
chief adaptation.
o Hence option (d) is the correct answer.
Q 48.B
• India is facing a land degradation crisis, more than 30 percent of its land area has been degraded through
deforestation, over-cultivation, soil erosion and depletion of wetlands.
• Due to desertification, researchers warned at the 14th meeting of the Conference of Parties to the United
Nations Convention to Combat Desertification (UNCCD COP 14), at least three to four species of
animals, such as the Indian Cheetah, pink-headed duck, and the Great Indian Bustard, have become
extinct.
• The Indian/Asiatic cheetah has been listed as Critically Endangered on the IUCN Red List since 1996
and now seems to have become extinct. Reduced gazelle numbers, persecution, land-use change, habitat
degradation and fragmentation, and desertification contributed to the decline of the cheetah population.
• Pink-headed ducks were one of the rarest species of waterfowl that were found in India. They were
usually found on enclosed waters surrounded by dense vegetation. They often inhabited small ponds that
were surrounded by bushes and high grass.
• The great Indian bustard inhabits dry grasslands and scrublands on the Indian subcontinent; its largest
populations was found in the Indian state of Rajasthan.
• Kharai Camels: found only in Kutch, are known to feed on mangroves on the island off shore. And to eat
this salty marine food, they sometimes swim for hours. However, with the destruction of mangroves in
Gujarat there are barely about 3,000 of these unique camels, that live in the mangroves, left in the
country. The government of India has declared them “highly threatened” and has launched a massive
conservation programme to increase Kharai numbers. Hence the correct answer is option (b).
Q 49.A
• Recent Context - The Reserve Bank of India (RBI) has said that the proposed Positive Pay System for
Cheque Truncation System will be introduced from January 1, 2021.
• Positive Pay Mechanism
o At present the Cheque Truncation System (CTS) for clearing cheques is operational pan-India. To
further augment customer safety in cheque payments and reduce instances of fraud occurring on
account of tampering of cheque leaves, RBI has decided to introduce a mechanism of Positive Pay.
The system adds an extra layer of security to reduce cheque-related frauds. Hence statement 1 is
correct.
o The concept of Positive Pay involves a process of reconfirming key details of large value cheques.
Under this process, the issuer of the cheque submits electronically, through channels like SMS,
mobile app, internet banking, ATM, etc., certain minimum details of that cheque (like date, name of
the beneficiary/ payee, amount, etc.) to the drawee bank, details of which are cross checked with the
presented cheque by CTS.
• National Payments Corporation of India (NPCI) shall develop the facility of Positive Pay in CTS and
make it available to participant banks.
• Banks shall enable it for all account holders issuing cheques for amounts of ₹50,000 and above. While
availing of this facility is at the discretion of the account holder, banks may consider making it mandatory
in case of cheques for amounts of ₹5,00,000 and above. Hence statement 2 is not correct.
Q 50.C
• Betla National Park: The Betla National Park, was first established as a sanctuary and later upgraded to
it’s present status. Average elevation is about 1000 feet .
o Animals like Gour, Langur, Tiger, Panther, Sloth and Wild Bear, Sambhar, Nilgai, Kakar, Mouse
Deer, etc are found there.
o Betla features waterfalls and natural hot springs on one hand, and historical monuments including a
16th century fort of Chero kings on the other. The North Koel River and its tributaries flow through
the northern portion of the park. Hence pair 1 is correctly matched.
• Papikonda National Park is located near Rajamahendravaram in the Papi Hills in East Godavari and
West Godavari districts of Andhra Pradesh.
20 www.visionias.in ©Vision IAS
o It is an Important Bird and Biodiversity Area and home to some endangered species of flora and
fauna.
o Godavari River flows through the park. Hence pair 2 is not correctly matched.
• Bhagwan Mahaveer Sanctuary and Mollem National Park is a protected area located in the Western
Ghats of West India, in Sanguem taluk, Goa State, along the eastern border with Karnataka.
o National Highway 4A divides it into two parts.
o It is home to waterfalls, such as Dudhsagar Falls and Tambdi Falls.
o The parkland is also home to a community of nomadic buffalo herders known as the Dhangar.
o The Mhadei River (known as Mondovi River downstream) flows through the sanctuary. Hence pair
3 is correctly matched.
o Vazra Sakla waterfalls is located in the sanctuary.
• Mukundra Tiger Reserve, popularly known as Darrah Wildlife Sanctuary, is spread across 4 districts
– Bundi, Kota, Jhalawar & Chittorgarh.
o It is the third tiger reserve in Rajasthan, after Ranthambore & Sariska Tiger Reserves.
o It is located on the eastern bank of the Chambal River and is drained by its tributaries. Hence pair 4
is not correctly matched.
Q 51.A
• An algal bloom is the rapid increase of the algae population in aquatic ecosystems, both in fresh and
marine waters, where it is known as either water bloom or marine bloom respectively. The major types of
algal blooms are cyanobacteria (blue-green algae) and red tides (red algal blooms).
• The development and proliferation of algal blooms likely result from a combination of environmental
factors including available nutrients, temperature, sunlight, ecosystem disturbance (stable/mixing
conditions, turbidity), hydrology (river flow and water storage levels) and the water chemistry (pH,
conductivity, salinity, carbon availability).
• Nutrients: Nutrients promote and support the growth of algae and Cyanobacteria. The eutrophication
(nutrient enrichment) of waterways is considered as a major factor. The main nutrients contributing to
eutrophication are phosphorus and nitrogen. In the landscape, runoff and soil erosion from fertilized
agricultural areas and lawns, erosion from river banks, river beds, land clearing (deforestation), and
sewage effluent are the major sources of phosphorus and nitrogen entering water ways. All of these are
considered as external sources. Hence option 1 is correct.
• Temperature: Early blue–green algal blooms usually develop during the spring when water
temperature is higher and there is increased light. The growth is sustained during the warmer months
of the year. Water temperatures above 25°C are optimal for the growth of Cyanobacteria. At these
temperatures, blue–green algae have a competitive advantage over other types of algae whose optimal
growth temperature is lower (12-15°C). In temperate regions, blue–green algal blooms generally do not
persist through the winter months due to low water temperatures. Higher water temperatures in tropical
regions may cause blue–green algal blooms to persist throughout the year. Hence option 2 is correct.
• Light: Blue–green algae populations are diminished when they are exposed to long periods of high
light intensity (photo-inhibition) but have optimal growth when intermittently exposed to high light
intensities. These conditions are met under the water surface where light environment is
fluctuating. Hence, option 3 is not correct.
• Stable Conditions: Most of blue–green algae prefer stable water conditions with low flows, long
retention times, light winds and minimal turbulence; other prefer mixing conditions and turbid
environments. Drought, water extraction for irrigation, human and stock consumption and the regulation
of rivers by weirs and dams all contribute to decreased flows of water in our river systems. Water moves
more slowly or becomes ponded, which encourages the growth of algae.
o In water bodies, another consequence of stable conditions is thermal stratification. Thermal
stratification occurs when the top layer of the water column becomes warmer and the lower
layer remains cooler. When the two layers stop mixing, the upper layer becomes more stable (no
wind-induced mixing, convection cells) and summer blooms of buoyant blue-green algae are
supported. Hence, option 4 is correct.
o When a water body is stratified, bottom waters often become depleted with oxygen (anoxia) which
may lead to increased nutrient release from the sediments. Pulses of nutrient from the colder bottom
layer may fuel up the algal growth in the top layer.
• Turbidity: Turbidity is caused by the presence of suspended particles and organic matter (flocs) in the
water column. High turbidity occurs when a lot of water is running through the system (high discharge
after a rain event). Low turbidity occurs when there is only a small amount of suspended matter present in
the water column. Low turbidity can be due to slow moving or stagnant water that allows suspended
articles to settle out of the water column. When turbidity is low, more light can penetrate through the
21 www.visionias.in ©Vision IAS
water column. This creates optimal conditions for algal growth. In return, growing algae create a
turbid environment"
Q 52.C
• A biodiversity hotspot is a biogeographic region with significant levels of biodiversity that is
threatened by human habitation.
• Initially, 25 biodiversity hotspots were identified but subsequently, eleven more have been added to the
list, bringing the total number of biodiversity hotspots in the world to 36.
• These hotspots are also regions of accelerated habitat loss.
• Although all the biodiversity hotspots put together cover less than 2 per cent of the earth’s land area, the
number of species they collectively harbour is extremely high and strict protection of these hotspots
could reduce the ongoing mass extinctions by almost 30 per cent.
• A region to get the status of a biodiversity hotspot has to have the following two criteria
o Endemism – At Least 1,500 species of its vascular plants must not be found anywhere else on the
Earth. Such unique species of plants are known as endemic plants or native to that particular region or
place. These plants are almost impossible to replace if lost completely.
o Habitat Loss – If around 70% of the native plantation of the region is lost, then it is deemed to be a
Biodiversity Hotspot.
• The biodiversity hotspots in India are Himalayas, Indo-Burma and the Western Ghats. Hence
option (c) is the correct answer.
Q 53.C
• Wildlife Protection Act,1972
• Sanctuary:
• Sanctuary is an area declared under section 18 or 38 or deemed under subsection 3 of section 36 to be
declared as a wildlife sanctuary. If the central government is satisfied that the conditions specified under
section 18 are fulfilled, the area by notification is declared as a sanctuary.
• The following are restricted inside the sanctuary.
• Entry inside the sanctuary is restricted one. Damage to boundary work is restricted. But the Chief Wildlife
Warden can permit the entry inside the sanctuary for the purpose of photography, scientific research,
tourism and transaction of any useful business with a person residing inside the sanctuary. Hence
statement 1 is correct.
• Destruction of forest property, causing fire, use of rigorous substances, grazing, possession of arms inside
the sanctuary is prohibited. Hence statement 2 is not correct.
• All livestock in and around the sanctuary region should be properly immunized. Hence statement 3 is
correct.
Q 54.B
• Bhitarkanika National Park : It is a large national park in northeast Kendrapara district in Odisha in
eastern India. The area is also been designated as second Ramsar site of the State after the Chilika
Lake. It is surrounded by Bhitarkanika Wildlife Sanctuary. Gahirmatha Beach and Marine
Sanctuary are to the east, separating swamp region and mangroves from the Bay of Bengal.
o The national park and wildlife sanctuary is inundated by the rivers Brahmani, Baitarani, Dhamra,
Pathsala. It hosts many mangrove species, and is the second largest mangrove ecosystem in India.
o The national park is home to Saltwater crocodile (Crocodylus porosus), Indian python, king cobra,
black ibis, darters and many other species of flora and fauna.
• Coringa Wildlife Sanctuary: It is a wildlife sanctuary and estuary situated near Kakinada in Andhra
Pradesh.
o It is the second largest stretch of mangrove forests in India with 24 mangrove tree species and more
than 120 bird species.
o It is home to the critically endangered white-backed vulture and the long billed vulture. In a
mangrove ecosystem the water bodies of the ocean/sea and the river meet together at a certain point.
• Pichavaram Bird Sanctuary: It is located in Cuddalore District, Tamil Nadu .It is about 190 km south
of Chennai.Pichavaram consists of a number of islands interspersing a vast expanse of water covered
with mangrove forest.
o According to a study, Pichavaram — the second largest mangrove wetland eco-system in the country
and an important area for migratory birds .It is separated from the Bay of Bengal by a sand bar. The
biotope consists of species like Avicennia and Rhizophora. It also supports the existence of rare
varieties of economically important shell and finfishes.
• Point Calimere Wildlife and Bird Sanctuary:It is a protected area in Tamil Nadu, along the Palk Strait
where it meets the Bay of Bengal at Point Calimere at the southeastern tip of Nagapattinam District. The
22 www.visionias.in ©Vision IAS
sanctuary was created for conservation of the least concern blackbuck antelope, an endemic mammal
species of India.
o It is famous for large congregations of waterbirds, especially greater flamingos. This site is a mix of
salt swamps, mangroves, backwaters, mudflats,rasslands and tropical dry evergreen forests.
Q 55.A
• India boasts of four biodiversity hotspots with many endemic bird species. Of the four, the Western Ghats
holds 29 endemic birds

Join us at Telegram
23 www.visionias.in Search → Upsc©Vision
4 EveryOne
IAS
Q 56.B
• Recent Context - The number of forest fires in the Pantanal wetland, the world's largest tropical
wetlands, has tripled in 2020 compared to last year, according to Brazil's national space agency.
• Pantanal Wetlands have located in South America Region. The wetlands are located across Brazil,
Paraguay, and Bolivia and are one of the most biodiverse areas in the world. Hence option (b) is
correct.
• It is the world’s largest tropical wetland. Wetlands like it are the Earth’s most effective carbon sinks,
which absorb and store more carbon than they release. Thus, they play a key role in mitigating climate
change. At roughly 200,000 square kilometers, the Pantanal comprises about 3% of the globe's
wetlands and plays a key role in the carbon cycle. When these carbon-rich ecosystems burn, vast amounts
of heat-trapping gases are released back into the atmosphere, contributing to the greenhouse effect.
• Brazil's National Institute of Space Research (INPE) has detected more than 21,200 fires in the Pantanal
biome so far this year, a figure that is already 69% higher than the full-year record from 2005 when INPE
recorded roughly 12,500 fires. There were 8,106 fires in September 2020 alone -- more than four times the
historic average for the month. The unusually dry conditions during this year have seen the blazes spread
further and faster because there were fewer natural water barriers. Climate change is just one part of the
problem. Large-scale deforestation in the Amazon rainforest to the north and the Cerrado savanna to the
east are also having profound effects on the Pantanal.
• The area is home to thousands of endangered or unusual species, including jaguars, capybaras, black
caimans, giant otters and hyacinth macaws. It's also an important stop on the routes of around 180 species
of migratory birds. According to the World Wide Fund for Nature, the Pantanal boasts the greatest
concentration of wildlife in South America.
• The fires ripping through the Pantanal are an example of a natural disaster that is exacerbated by climate
change while simultaneously making the problem worse. Extreme weather events, such as drought and
floods, are becoming more frequent and more severe around the world, and the Pantanal is no exception.
There are indications that the region is getting drier and warmer as the global temperatures rise.

Q 57.C
• Transportation is the process of transporting water, minerals and food to all parts of the plant
body. Transport in plants occurs at three levels: The uptake and release of water and solute by individual
cells. Short distance transport of substances from one cell to another. Long-distance transport of sap
within xylem and phloem.
• There are two means of transport in plants.
o Diffusion: Diffusion is a passive process of transport. A single substance tends to move from an area
of high concentration to an area of low concentration until the concentration is equal across
space. Diffusion expends no energy. On the contrary, concentration gradients are a form of
potential energy, dissipated as the gradient is eliminated. Hence statement 3 is correct.
o Active Transport: Active transport use energy in the form of ATP in the process of pumping
molecules against the concentration gradient.
• The xylem and the phloem make up the vascular tissue of a plant and transport water, sugars, and other
important substances around a plant.
• Xylem is the tissue responsible for supporting the plant as well as for the storage and long-
distance transport of water and nutrients, including the transfer of water-soluble growth factors from
the organs of synthesis to the target organs. Xylem is present at the centre of vascular bundles where the
transport of water and mineral is unidirectional. so when Xylem tissue is mechanically blocked,
the transport of water and nutrients is severely affected. Hence statement 2 is not correct.
• Phloem is a vascular tissue that transports soluble organic compounds prepared during photosynthesis
from the green parts of the plant to the rest of the plant. Phloem tissue is present towards the periphery of
the vascular bundles. The transport in the phloem is bidirectional where the food can move both up and
down the tissues. Hence statement 1 is not correct.
• Hence option (c) is the correct answer.

Q 58.C
• On the basis of nutrient enrichment, the lakes are of the following types:
o Oligotrophic: An oligotrophic lake is one which has relatively low productivity due to the low
nutrient content in the lake. These lakes have low algal production, and consequently, often have
very clear waters, with high drinking-water quality. The bottom waters of such lakes typically
have ample oxygen; thus, such lakes often support many fish species such as lake trout, which

24 www.visionias.in ©Vision IAS


require cold, well-oxygenated waters. These lakes are usually found in the cold regions of the world
where mixing of nutrients is rare.
o Mesotrophic: Lakes with an intermediate level of productivity are called mesotrophic lakes. These
lakes are usually clear water with submerged aquatic plants.
o Eutrophic: Lakes have high levels of biological productivity due to high concentration of nutrients,
especially nitrogen and phosphorus.
o Hence option (c) is correct.

Q 59.D
• Cestrum nocturnum commonly known as the queen of the night is a popular ornamental species
widely distributed for its strongly fragrant flowers. It is known to occur in natural forests, planted forests,
riparian zones, ruderal or disturbed, shrublands, and in urban areas. It is often cultivated as an ornamental
plant in gardens. It is able to escape and can establish dense, impenetrable thickets in the scrub, moist or
wet forests including riparian zones.
• Cestrum nocturnum produces long-lived and widely dispersed seeds and forms dense, shady masses. It is
most likely to invade disturbed and open forest and margins, streamsides, and shrublands, especially in
warmer areas.
• Recently it is reported that the spread of Cestrum nocturnum is posing a threat to all Shola and
grassland habitats as it does not allow any native flora to thrive. The Cestrum plants need to
be completely removed with their roots, otherwise, they will keep sprouting and keep taking over native
grassland ecosystem.
• Hence option (d) is the correct answer.

Q 60.A
• In India, ecologically unique and biodiversity-rich regions are legally protected as biosphere reserves,
national parks and sanctuaries. India now has 14 biosphere reserves, 90 national parks and 448
wildlife sanctuaries.
• India has also a history of religious and cultural traditions that emphasised the protection of nature. In
many cultures, tracts of forest were set aside, and all the trees and wildlife within were venerated
and given total protection.
• Such sacred groves are found in Khasi and Jaintia Hills in Meghalaya, Aravalli Hills of Rajasthan,
Western Ghat regions of Karnataka and Maharashtra and the Sarguja, Chanda and Bastar areas of Madhya
Pradesh. In Meghalaya, the sacred groves are the last refuges for a large number of rare and
threatened plants as an in-situ method of conservation.
• In Ex-situ Conservation approach, threatened animals and plants are taken out from their natural habitat
and placed in a special setting where they can be protected and given special care. Zoological
parks, botanical gardens and wildlife safari parks serve this purpose.
• Hence option (a) is the correct answer.

Q 61.C
• Bioluminescence is the property of a living organism to produce and emit light. Hence statement 1 is
correct.
• Bioluminescent organisms are usually found in ocean environments, but they are also found on terrestrial
environments. Hence statement 2 is correct.
• The colour of the light emitted by the organism depends on its chemical properties. In the case of fungi,
the luminescence comes from the enzyme, luciferase. The light emits when luciferans is catalysed by the
enzyme luciferase, in the presence of oxygen. During the chemical reaction, several unstable intermediate
products are released as excess energy that makes them visible as light.
• A new mushroom species ‘Roridomyces phyllostachydis’ was discovered in the forests of Meghalaya
— glow bright green. It was first sighted on a wet August night near a stream in Meghalaya’s
Mawlynnong in East Khasi Hills district and later at Krang Shuri in West Jaintia Hills district. It is now
one among the 97 known species of bioluminescent fungi in the world. Hence option (c) is the correct
answer.

Q 62.C
• Recently, a three-decade-old unresolved ethno-territorial conflict between Armenia and Azerbaijan over
Nagorno-Karabakh flared up once again. The conflict is between two relatively small countries and is
territorial in nature. However, several regional and global players particularly Russia, Europe, Turkey, and
Iran are also involved to secure their strategic, security and economic interests in the region.
25 www.visionias.in ©Vision IAS
• OSCE Minsk Group was created in 1992 by the Conference on Security and Cooperation in Europe
(CSCE) to encourage a peaceful, negotiated resolution to the conflict between Azerbaijan and
Armenia over Nagorno-Karabakh.
o Nagorno-Karabakh, the center of the conflict, is located within Azerbaijan but is populated, mostly,
by those of Armenian ethnicity (and mostly Christian compared to the Shia Muslim majority
Azerbaijan).
o OSCE is the world's largest security-oriented intergovernmental organization. Its mandate includes
issues such as arms control, promotion of human rights, freedom of the press, and fair elections.
o The Organisation for Security and Co-operation in Europe (OSCE) Minsk Group, chaired by France,
Russia, and the USA, has tried to get the two countries to reach a peace agreement for several years.

Q 63.C
• Recent context - The Foreign Contribution (Regulation) Amendment Bill, 2020 has been passed by
parliament.
• The Parliament has passed The Foreign Contribution (Regulation) Amendment Bill, 2020. It
amends Foreign Contribution (Regulation) Act, 2010 (FCRA). The Act regulates the acceptance and
utilisation of foreign contribution by individuals, associations and companies.
• The amendments were introduced to FCRA
o To regulate non-governmental organisations and make them more accountable and transparent.
o To regulate religious conversions, which are supported by foreign funds.
o To ensure foreign money is not used against national interests or for anti-national activities
• Provisions of the Amendment
o Prohibition to accept foreign contribution: Under the Act election candidates, editor or publisher of
a newspaper, judges, government servants, members of any legislature are prohibited to accept any
foreign contribution. The Amendment adds public servants to this list. Hence statement 2 is correct.
o Transfer of foreign contribution: Under the Act, foreign contribution cannot be transferred to any
other person unless such person is also registered to accept foreign contribution. The Amendment
prohibits the transfer of foreign contribution to any other person.
o Aadhaar, passport and OCI card for registration: Amendment adds that any person seeking prior
permission, registration must provide the Aadhaar number of all its office bearers, directors etc. In
case of a foreigner, they must provide a copy of the passport or the Overseas Citizen of India card for
identification.
o FCRA account: Amendment states that foreign contribution must be received only in an account
designated by the bank as ‘FCRA account’ in such branch of the State Bank of India, New Delhi, as
notified by the central government. No funds other than the foreign contribution should be received or
deposited in this account. The person may open another FCRA account in any scheduled bank of their
choice for keeping or utilising the received contribution.

26 www.visionias.in ©Vision IAS


o Restriction in the utilisation of foreign contribution: Under the Act, if a person accepting foreign
contribution is found guilty of violating any provisions of the Act, the unutilised foreign contribution
may be utilised, only with the prior approval of the central government. The Amendment adds that the
government may also restrict usage of unutilised foreign contribution for such persons based on a
summary inquiry, and pending any further inquiry.
o Renewal of license: Under the Act, every person who has been given a certificate of registration must
renew the certificate within six months of expiration. Amendment adds that the government may
conduct an inquiry before renewing the certificate to ensure that the person has fulfilled all conditions
specified in Act.
o Reduction in use of foreign contribution for administrative purposes: Under the Act, a person
who receives foreign contribution must use it only for the purpose for which the contribution is
received. Further, they must not use more than 50% of the contribution for meeting administrative
expenses. Amendment reduces this limit to 20%. Hence statement 1 is correct.
o Suspension of registration: Under the Act, the government may suspend the registration of a person
for a period not exceeding 180 days. Amendment adds that such suspension may be extended up to an
additional 180 days.

Q 64.B
• Recent Context - Due to restrictions caused by COVID-19, Pulikkali went digital.
• Pullikalli – Pulikkali [Puli (Tiger) Kali-(Play)] is a recreational street folk art from the state of Kerala. It
is mainly practised in Thrissur district of Kerala. It is part of Onam celebrations in Kerala. Hence option
(b) is the correct answer.
• It is a recreational street folk art performed on the fourth day of Onam celebrations.
• The main theme of this folk art is tiger hunting with participants playing the role of tiger and hunter.
Performers paint their bodies like tigers and hunters and dance on streets to the beats of traditional
percussion instruments such as thakil, udukku and chenda. It is also known as Tiger Dance.
• It was introduced by Maharaja Rama Varma Sakthan Thampuran, the then Maharaja of Cochin.
• Onam Festival - Onam (August–September) is a Hindu rice harvest festival of Kerala celebrated to
commemorate King Mahabali. According to a popular legend, the festival is celebrated to welcome King
Mahabali, whose spirit is said to visit Kerala at the time of Onam. During the festival, people get dressed
in their traditional attire, prepare the Onam Sadhya and take part in a variety of group activities such as
the Pookolam (rangoli made with fresh flowers), Vallam Kali (boat race), Pulikali (tiger dance), Kai Kottu
Kali (Onam dance), Kummattikali (mask dance) and various other activities.

Q 65.D
• Homeostasis is the state of steady internal chemical and physical conditions maintained by living
systems. It is referred to as the mechanism to maintain a stable internal environment instead of
changes taking place in the external environment.
• Some organisms are able to maintain homeostasis by the following process:
• Regulate:
o Some organisms are able to maintain homeostasis by physiological (sometimes behavioural also)
means which ensures constant body temperature, constant osmotic concentration, etc. The
mechanisms used by most mammals to regulate their body temperature are similar to the ones that we
humans use.
o We maintain a constant body temperature of 37 degrees C. In summer, when the outside temperature
is more than our body temperature, we sweat profusely and thus bring down the body temperature.
• Conform: An overwhelming majority (99 per cent) of animals and nearly all plants cannot maintain a
constant internal environment. Their body temperature changes with the ambient temperature. In aquatic
animals, the osmotic concentration of the body fluids changes with that of the ambient air, water osmotic
concentration. These animals and plants are simply conformers. Examples: Lizards, insects and fish.
• Migrate: The organism can move away temporarily from the stressful habitat to a more hospitable area
and return when the stressful period is over. Example: Every winter the famous Keoladeo National Park
(Bharatpur) in Rajasthan host thousands of migratory birds coming from Siberia and other extremely cold
northern regions.
• Suspend: In bacteria, fungi and lower plants, various kinds of thick-walled spores are formed which help
them to survive unfavourable conditions – these germinate on availability of suitable environment.
Example: bears going into hibernation during winter is an example of an escape in time. Some snails and
fish go into aestivation to avoid summer-related problems-heat and desiccation. Hence option (d) is the
correct answer.
27 www.visionias.in ©Vision IAS
Q 66.C
• Temperature is the most important ecologically relevant environmental factor. The average temperature
on land varies seasonally, decreases progressively from the equator towards the poles and from plains to
the mountain tops. It ranges from subzero levels in polar areas and high altitudes to >500C in tropical
deserts in summer. There are, however, unique habitats such as thermal springs and deep-sea
hydrothermal vents where average temperatures exceed 1000 C.
• Temperature affects the kinetics of enzymes and through it the metabolic activity and other physiological
functions of the organism. A few organisms like Plants- Roses, daisies, some vegetables and fruits etc.
Animals- Cat, tiger, dog etc. can tolerate and thrive in a wide range of temperatures (they are called
eurythermal ).
• A vast majority of them are restricted to a narrow range of temperatures (such organisms are
called stenothermal) like Eg: Plants- Bougainvillaea, orchids, frangipani, some fruits, Croton
etc. Animals- Penguin, Python, Crocodile. The levels of thermal tolerance of different species
determine to a large extent their geographical distribution. Hence both statement 1 and statement 2 are
correct.

Q 67.A
• Recent Context - The Customs (Administration of Rules of Origin under Trade Agreements) Rules, 2020
(CAROTAR, 2020), shall come into force from 21st September 2020.
• CAROTAR, 2020 aims to protect the domestic industry from misuse of Free Trade Agreements
(FTAs). The Union Finance Minister in the Budget 2020 has said -“Undue claims of FTA benefits have
posed threat to the domestic industry. Such imports require stringent checks. In this context, suitable
provisions are being incorporated in the Customs Act, 1962.” CAROTAR, 2020 are a set of guidelines for
enforcement of the ‘rules of origin’ for allowing preferential rate on imports under free trade
agreements. Hence option (a) is the correct answer.
• It aims to supplement the existing operational certification procedures prescribed under different trade
agreements.
• It requires an importer to conduct a basic level of due diligence before importing the goods and satisfy
himself that the goods meet the prescribed originating criteria. For this purpose, a list of minimum
information which the importer is required to possess while importing the goods has been provided in the
rules along with general guidance. This will support the importer to correctly ascertain the country of
origin, properly claim the concessional duty and assist customs authorities in ensuring smooth clearance
of legitimate imports. Also, an importer would now have to enter certain origin related information in the
Bill of Entry, as available in the Certificate of Origin. The new rules would strengthen the hands of the
Customs in checking any attempted misuse of the duty concessions under FTAs.

Q 68.C
• The ocean absorbs vast quantities of heat as a result of increased concentrations of greenhouse gases in
the atmosphere, mainly from fossil fuel consumption.
• The Fifth Assessment Report published by the Intergovernmental Panel on Climate Change (IPCC) in
2013 revealed that the ocean had absorbed more than 93% of the excess heat from greenhouse gas
emissions since the 1970s. This is causing ocean temperatures to rise. The distribution of excess heat in
the ocean is not uniform, with the greatest ocean warming occurring in the Southern Hemisphere and
contributing to the subsurface melting of Antarctic ice shelves. Hence, statement 1 is correct.
• Increasing ocean temperatures affect marine species and ecosystems. Rising temperatures cause coral
bleaching and the loss of breeding grounds for marine fishes and mammals. Rising ocean temperatures
also affect the benefits humans derive from the ocean – threatening food security, increasing the
prevalence of diseases and causing more extreme weather events and the loss of coastal protection.
• From 2004-2013, seawater became increasingly salty in the western Indian Ocean and near the equator in
the western and central tropical Pacific, as well as in the high evaporation areas of the eastern subtropical
Pacific in both hemispheres. Meanwhile, much of the North Atlantic and eastern tropical South Pacific is
freshening.
• Studies incorporating both observations and climate model simulations of global warming indicate that
over time, precipitation will increase in rainy areas and evaporation will increase in dry areas, making
fresh areas of the ocean fresher and salty areas of the ocean saltier. Hence, statement 2 is correct.
• Achieving the mitigation targets set by the Paris Agreement on climate change and limiting the global
average temperature increase to well below 2°C above pre-industrial levels is crucial to prevent the
massive, irreversible impacts of ocean warming on marine ecosystems and their services.

28 www.visionias.in ©Vision IAS


• Establishing marine protected areas and putting in place adaptive measures, such as precautionary catch
limits to prevent overfishing, can protect ocean ecosystems and shield humans from the effects of ocean
warming.

Q 69.D
• Wetlands are among the world’s most productive environments. They are wellsprings of biological
diversity, providing the water and primary productivity upon which countless species of plants and
animals depend for survival. They support high concentrations of birds, mammals, reptiles, amphibians,
fish and invertebrate species.
• Wetlands are also important storehouses of plant genetic material. Rice, for example, which is a
common wetland plant, is the staple diet of more than half of humanity.
• Wetlands provide a multitude of benefits, including filtering our water, ensuring biodiversity, protecting
our coastlines, and mitigating climate change.
• The interactions of physical, biological and chemical components of a wetland, as part of the “natural
infrastructure” of the planet, such as soils, water, plants and animals, enable the wetland to perform many
vital functions, for example, water storage; storm protection and flood mitigation; drought buffering;
shoreline stabilization and erosion control; groundwater recharge and discharge; water purification;
retention of nutrients, sediments, and pollutants; and stabilization of local climate conditions, particularly
rainfall and temperature
• Wetlands continuously remove and store atmospheric carbon. Plants take it out of the atmosphere and
convert it into plant tissue, and ultimately into soil when they die and decompose. At the same time,
microbes in wetland soils release greenhouse gases into the atmosphere as they consume organic matter.
• Natural wetlands typically absorb more carbon than they release. But as the climate warms wetland soils,
microbial metabolism increases, releasing additional greenhouse gases. In addition, draining or disturbing
wetlands can release soil carbon very rapidly.
• For these reasons, it is essential to protect natural, undisturbed wetlands. Wetland soil carbon,
accumulated over millennia and now being released to the atmosphere at an accelerating pace, cannot be
regained within the next few decades, which are a critical window for addressing climate change. In some
types of wetlands, it can take decades to millennia to develop soil conditions that support net carbon
accumulation. Other types, such as new saltwater wetlands, can rapidly start accumulating carbon.
• Hence all the options are correct.

Q 70.C
• A swamp is a wetland that is forested. Many swamps occur along large rivers where they are critically
dependent upon natural water level fluctuations. Other swamps occur on the shores of large.
• Swamp ecosystem is characterized by plant life dominated by trees. This characteristic distinguishes a
swamp from a marsh, in which plant life consists largely of grasses. Hence, statement 2 is not correct.
• Swamps are found throughout the world. They exist in areas with poor drainage and sufficient water
supply to keep the ground waterlogged, and they have a high enough supply of minerals in the water to
stimulate decay of organisms and prevent the accumulation of organic materials.
• Swamps are characterized by mineral soils and exist in areas with poor drainage and sufficient water
supply to keep the ground waterlogged, and they have a high enough supply of minerals in the water to
stimulate decay of organisms and prevent the accumulation of organic materials. The largest swamp in
the world is the Amazon River floodplain, which is particularly significant for its large number of fish
and tree species. Hence statements 1 and 3 are correct.
Q 71.D
• Seaweeds are (thalloid plants) macroscopic algae, which mean they have no differentiation of true tissues
such as roots, stems and leaves. They have leaf-like appendages.
• Seaweeds, the larger and visible marine plants are found attached to rocks, corals and other submerged
strata in the intertidal and shallow subtidal zones of the sea. Seaweeds grow in shallow coastal waters
wherever sizable substrata are available.
• Uses of seaweeds:
o Seaweeds are important as food for humans, feed for animals, and fertilizer for plants. Seaweed is
chock-full of vitamins, minerals, and fibre, and can be tasty.
o Seaweeds are used as a drug for goitre treatment, intestinal and stomach disorders. Many seaweeds
contain anti-inflammatory and anti-microbial agents.
o Products like agar-agar and alginates, iodine which are of commercial value, are extracted from
seaweeds.

29 www.visionias.in ©Vision IAS


o By the biodegradation of seaweeds methane like economically important gases can be produced
in large quantities. The U.S. Department of Energy is investing in a novel source of future
renewable energy in the form of seaweed.
o Seaweeds are also used as the potential indicators of pollution in the coastal ecosystem, particularly
heavy metal pollution due to their ability to bind and accumulate metals strongly.
o Among their many uses in manufacturing, they are effective binding agents (emulsifiers) in such
commercial goods as toothpaste and fruit jelly, and popular softeners (emollients) in organic
cosmetics and skin-care products.
o Hence, option (d) is correct.

Q 72.B
• Statement 1 is correct: The Pradhan Mantri Matsya Sampada Yojana (PMMSY) is a flagship scheme for
focused and sustainable development of fisheries sector in the country with an estimated investment of Rs.
20,050 crores for its implementation during a period of 5 years from FY 2020-21 to FY 2024-25 in all
States/Union Territories, as a part of AatmaNirbhar Bharat Package.
• Statement 2 is not correct: The investment of Rs. 20,050 crores under PMMSY is the highest ever in the
fisheries sector. Out of this, an investment of about Rs 12340 crores is proposed for beneficiary-oriented
activities in Marine, Inland fisheries and Aquaculture and about Rs 7710 crores investment for Fisheries
Infrastructure.
• Statement 3 is correct: PMMSY aims at enhancing fish production by an additional 70 lakh tonne by
2024-25, increasing fisheries export earnings to Rs.1,00,000 crore by 2024-25, doubling of incomes of
fishers and fish farmers, reducing post-harvest losses from 20-25% to about 10% and generation of
additional 55 lakhs direct and indirect gainful employment opportunities in the fisheries sector and allied
activities.

Q 73.B
• Ramsar sites are wetlands of international importance designated under Ramsar Convention. To be
designated as a Ramsar site, the nominated wetland must fulfil at least one of the nine criteria designated
by the Ramsar Convention.
• Group A of the Criteria: Sites containing representative, rare or unique wetland types
o Criterion 1: A wetland should be considered internationally important if it contains a representative,
rare, or unique example of a natural or near-natural wetland type found within the appropriate
biogeographic region.
• Group B of the Criteria. Sites of international importance for conserving biological diversity
• Criteria based on species and ecological communities
o Criterion 2: A wetland should be considered internationally important if it supports vulnerable,
endangered, or critically endangered species or threatened ecological communities. Hence,
option (a) is correct.
o Criterion 3: A wetland should be considered internationally important if it supports populations of
plant and/or animal species important for maintaining the biological diversity of a particular
biogeographic region.
o Criterion 4: A wetland should be considered internationally important if it supports plant
and/or animal species at a critical stage in their life cycles, or provides refuge during adverse
conditions.

• Specific criteria based on waterbirds


o Criterion 5: A wetland should be considered internationally important if it regularly supports
20,000 or more waterbirds. Hence, option (b) is not correct.
o Criterion 6: A wetland should be considered internationally important if it regularly supports 1% of
the individuals in a population of one species or subspecies of waterbird.
• Specific criteria based on fish
o Criterion 7: A wetland should be considered internationally important if it supports a significant
proportion of indigenous fish subspecies, species or families, life-history stages, species interactions
and/or populations that are representative of wetland benefits and/or values and thereby contributes to
global biological diversity.
o Criterion 8: A wetland should be considered internationally important if it is an important source of
food for fishes, spawning ground, nursery and/or migration path on which fish stocks, either within
the wetland or elsewhere, depend.
30 www.visionias.in ©Vision IAS
• Specific criteria based on other taxa
o Criterion 9: A wetland should be considered internationally important if it regularly supports
1% of the individuals in a population of one species or subspecies of wetland -dependent non -
avian animal species. Hence option (c ) is correct.

Q 74.D
• Total Maximum Daily Load ( TMDL ) is a calculation of the maximum amount of a pollutant that a
waterbody can receive and still meet water quality standards . Hence , option (d ) is the correct answer.
• In other words, it is the sum of the allowable loads of a single pollutant from all contributing point and
non -point sources and includes a margin of safety and consideration of seasonal variations . In addition , a
TMDL contains the reductions needed to meet water quality standards and allocates those reductions
among the sources in the watershed.
• TMDL determines a pollutant reduction target and allocates load reductions necessary to the source(s ) of
the pollutant .
• TMDLs can be calculated for correcting water pollution from specific point source discharges or
throughout a watershed and balance the location and amount of needed pollution reductions.
• The Kessler syndrome is a theoretical scenario in which the density of objects in low Earth orbit ( LEO )
due to space pollution is high enough that collisions between objects could cause a cascade in which each
collision generates space debris that increases the likelihood of further collisions .

Q 75 .D
• Plant hormones (also known as phytohormones) are organic substances that regulate plant growth and
development . Plants produce a wide variety of hormones, including auxins, gibberellins, abscisic acid ,
cytokinins, salicylic acid, ethylene , jasmonates, brassinosteroids, and peptides.
• Physiological effects of Plant Growth Regulators:
o Auxins: These are growth-promoting substances that contribute to the elongation of shoots. They help
to initiate rooting in stem cuttings, an application widely used for plant propagation. Auxins
promote flowering e.g . in pineapples. They help to prevent fruit and leaf drop at early stages but
promote the abscission of older mature leaves and fruits. Hence statement 1 is correct ,
o Gibberellins : They promote cell elongation , shoot growth . They cause an increase in the length of
the axis, a response which is widely used to increase the length of grapes stalks. Gibberellins, cause
fruits like apple to elongate and improve its shape. They also delay senescence. Hence statement 2 is
correct.
o Cytokinins : Cytokinins are synthesised in regions where rapid cell division occurs, for example , root
apices, developing shoot buds, young fruits etc. It helps to produce new leaves, chloroplasts in
leaves, lateral shoot growth and adventitious shoot formation. Cytokinins help overcome apical
dominance. They promote nutrient mobilisation which helps in the delay of leaf senescence.
o Ethylene : Ethylene promotes abscission of leaves and fruits, inhibits shoot elongation and inhibits
lateral bud development. In apples and cherries, ethylene is involved in the transition of fruit from
being physiologically mature to ripe . Hence statement 3 is correct.
o Abscisic Acid : Abscisic acid controls the dormancy of buds and seeds, inhibits shoot growth and is
involved in regulating water loss from plants.

Q 76 .C
• Dolphins are one of the oldest creatures in the world along with some species of turtles, crocodiles and
sharks. The Ganges river dolphin was officially discovered in 1801. Ganges river dolphins once lived in
the Ganges -Brahmaputra -Meghna and Karnaphuli-Sangu river systems of Nepal, India , and Bangladesh .
But the species is extinct from most of its early distribution ranges.
• WWF-India identified optimal habitats in 9 stretches in 8 rivers as ideal habitats for the Ganges river
dolphin population and hence for prioritized conservation action. These include: Upper Ganga River
( Brijghat to Narora ) in the state of Uttar Pradesh ( Proposed Ramsar Site ), Chambal River ( up to 10 km
downstream of Chambal Wildlife Sanctuary ) in the state of Madhya Pradesh and Uttar Pradesh , Ghagra
and Gandak River, in the state of Uttar Pradesh and Bihar, Ganga River, from Varanasi to Patna in Uttar
Pradesh and Bihar respectively, Son and Kosi River in Bihar, Brahmaputra River from Sadia ( foothills of
Arunachal Pradesh ) upto the Dhubri ( Bangladesh Border ), Kulsi River a tributary of Brahmaputra .
• They are locally they are known as the Susu. The name “Susu” is a reference to the sound they make
when coming to the surface to breathe.
• The Ganges river dolphin can only live in freshwater and is essentially blind . They hunt by emitting
ultrasonic sounds, which bounces off of fish and other prey , enabling them to “see” an image in their
31 ©Vision IAS
mind. They are frequently found alone or in small groups, and generally a mother and calf travel together .
Calves are chocolate brown at birth and then have grey - brown smooth, hairless skin as adults . Females are
larger than males and give birth once every two to three years to only one calf . Hence statement 1 and
statement 2 are correct .
• The maximum size of a female is 2.67 m and of a male 2.12 m . Females attain sexual maturity at an age
of 10 -12 years, while the males mature earlier. The gestation period is 9- 11 months and a female gives
birth to only one calf, once in 2- 3 years.
• Ganges river dolphin has dwindled abysmally to less than 2000 during the last century owing to direct
killing, habitat fragmentation by dams and barrages and indiscriminate fishing
• It is protected under the Indian Wildlife Act, although these legislations require stricter enforcement. Both
subspecies are listed by the IUCN as endangered on their Red List of Threatened Species.
Q 77 . B
• Spices and herbs are defined as plant-derived substances that add flavour to any dish . Spices can come
from the following plant parts : roots, rhizomes, stems, leaves, bark , flowers, fruits, and seeds. Herbs are
typically thought of as non-woody plants .
• Saffron is a spice derived from the flower of Crocus sativus, commonly known as the "saffron crocus".
The stigma and styles , called threads, are collected and dried for use mainly as a seasoning and colouring
agent in food . Hence pair 1 is not correctly matched .
• Turmeric is a spice that comes from the turmeric plant . It is commonly used to impart a distinct flavour
to food . As a dried rhizome of a herbaceous plant, turmeric is closely related to ginger . Turmeric is
used in a wide variety of foods of the cuisines of Southern Asia but locally it also applies as an antiseptic
for skin abrasions and cuts . Hence pair 2 is correctly matched.
• Cinnamon is a spice obtained from the inner bark of several tree species from the genus Cinnamomum .
Cinnamon is used mainly as an aromatic condiment and flavouring additive in a wide variety of
cuisines . Hence pair 3 is correctly matched .
• Other Spices and their sources in the plants:
Plant
Spice
Part
Seed Cumin , fenugreek , coriander
Bulb Onion , Garlic
Fruit Chilli, Cardamom
Leaf Mint
Rhizome Ginger, Turmeric
Bud Clove
Berry Black pepper
Q 78.C
Recent Context - A recent study published in the journal Scientific Reports has found that the tree
'African Baobab' has 168 chromosomes. Previous studies estimated that the tree has between 96 and 166
chromosomes. The study will be useful in genetic conservation efforts .
Baobabs are deciduous trees ranging in height from 5 to 20 meters . The African baobab ( Adansonia
digitata ) is one of the nine species of baobab .
Baobabs are trees recognisable by their distinctive swollen stems, where they store massive amounts of
water in their stems to cope with seasonal droughts. Hence option ( c ) is not correct Elephants .

sometimes tear the trees down to get to the moisture inside. The tree is drought, fire and termites resistant.
It is also called tree of life as African baobab is a succulent, which means that during the rainy season it
absorbs and stores water in its vast trunk , enabling it to produce a nutrient -dense fruit in the dry season
when all around is dry and arid . Hence option ( a ) is correct .
It is also called upside down trees because their branches look like roots. Hence option ( d ) is correct .
Baobab trees can live for more than a thousand years and provide food , livestock fodder, medicinal
compounds, and raw materials. When they die , they rot inside and suddenly collapse. They regrow their
bark if it is stripped . Hence option ( b) is correct .
As their flowers emerge in the evening, they are pollinated by bats and nocturnal insects such as moths .
Uses : Firewood, Building of structures, Medicinal, Food and Drink.
Since 2005 , 9 of the 13 oldest African baobab specimens and 5 of the 6 largest trees have died or suffered
the collapse and death of their largest or oldest stems, that may have been caused by the effects of climate
change .
32 ©Vision IAS
Q 79.C
• Bio-prospecting is a kind of commercial venture for utilizing plants or plant products in large-scale
communities as a usable commodity.
• The concept of ‘bioprospecting’ is based on recognition of the importance of natural product discovery for
the development of new crops and medicines, often based on traditional knowledge. For example, in many
developing countries, a large part of the population depends upon traditional medicines for their primary
health care needs.
• In India, 65% of the population only has access to traditional systems of medicine, and in Africa, 80% of
the population uses traditional medicines. Much of this knowledge has not been examined using the most
advanced scientific methods, however, this is rapidly changing.
• The need to incentivize the conservation of biodiversity through fair and equitable benefit sharing
saw the Convention on Biological Diversity (CBD) text include elements on ‘access and benefit-
sharing’ (ABS). Article 1 of the CBD lists its three objectives as:
o the conservation of biological diversity;
o the sustainable use of its components; and
o the fair and equitable sharing of benefits arising from such use.
• The CBD recognizes the sovereign rights of states over genetic resources found in situ conditions within
their territories. This is unprecedented in international law to the extent that it requires parties to the CBD
to uphold within their jurisdictions, the rights of other parties to the CBD over their (the latter’s) genetic
resources
• The Nagoya Protocol is an international treaty that was adopted in October 2010 by the 193 Parties to
the Convention on Biological Diversity (CBD) based on the third objective of CBD. The aim of the
Nagoya Protocol on Access and Benefit Sharing is to give effect to the fair and equitable benefit-
sharing provisions of the CBD. Hence the correct option is (c)

Q 80.C
• An important characteristic of all communities is that their composition and structure constantly change in
response to the changing environmental conditions. This change is orderly and sequential, parallel with
the changes in the physical environment. These changes lead finally to a community that is in near
equilibrium with the environment and that is called a climax community. The gradual and fairly
predictable change in the species composition of a given area is called ecological succession.
• Succession is hence a process that starts in an area where no living organisms are there – these could
be areas where no living organisms ever existed, say bare rock; or in areas that somehow, lost all the
living organisms that existed there. The former is called primary succession, while the latter is termed
secondary succession.
• The entire sequence of communities that successively change in a given area is called sere(s). In the
successive seral stages, there is a change in the diversity of species of organisms, an increase in
the number of species and organisms as well as an increase in the total biomass. Hence both
statements 1 and statement 2 are correct.

Q 81.C
• Recent Context - The Singapore Convention on Mediation has come into force.
• The Singapore Convention on Mediation (also known as the United Nations Convention on
International Settlement Agreements) is a uniform and efficient framework for international settlement
agreements resulting from mediation (‘settled agreement’). Hence option (c) is the correct answer.
• It applies to international settlement agreements resulting from mediation, concluded by parties to resolve
a commercial dispute. It will allow businesses to seek enforcement of a mediated settlement agreement
across borders by applying directly to courts of countries that have signed and ratified the treaty.
• Currently, the Convention has 53 signatories, including India, China, and the US.
• Significance:
o It will facilitate international trade and commerce by enabling disputing parties to easily enforce and
invoke settlement agreements across borders.
o Businesses will benefit from mediation as an additional dispute resolution option to litigation and
arbitration in settling cross-border disputes. It helps to save time and legal costs of companies.
o Being a binding international instrument, it is expected to bring certainty and stability to the
international framework on mediation, thereby contributing to the Sustainable Development Goals
(SDG), mainly the SDG 16.

33 www.visionias.in ©Vision IAS


Q 82.D
• The plant growth regulators (PGRs) are small, simple molecules of diverse chemical composition.
They could be indole compounds, adenine derivatives, derivatives of carotenoids, terpenes or gases
(ethylene). Hence option (d) is the correct answer.
• The PGRs can be broadly divided into two groups based on their functions in a living plant body. One
group of PGRs have involved in growth-promoting activities, such as cell division, cell enlargement,
pattern formation, tropic growth, flowering, fruiting and seed formation. These are also called plant
growth promoters, e.g., auxins, gibberellins and cytokinins.
• The PGRs of the other group play an important role in plant responses to wounds and stresses of biotic
and abiotic origin. They are also involved in various growth-inhibiting activities such as dormancy and
abscission. The PGR abscisic acid belongs to this group. The gaseous PGR, ethylene, could fit either of
the groups, but it is largely an inhibitor of growth activities.
Q 83.A
• Recent Context - Union Power Minister has recently launched Green Term Ahead Market (GTAM) in
electricity through video conferencing. Hence statement 2 is not correct.
• About GTAM – It is an alternative new model introduced for selling off the power by the renewable
developers in the open market without getting into long term Power Purchase Agreements (PPAs).
• GTAM will provide an exclusive platform for short-term trading of Renewable Energy. Hence statement
1 is correct.
• Key features of GTAM:
o Transactions through GTAM will be bilateral in nature with clear identification of corresponding
buyers and sellers, there will not be any difficulty in accounting for Renewable Purchase Obligations
(RPO).
o There will be separate contracts for both Solar and Non-Solar energy to facilitate Solar and Non-Solar
RPO fulfilment.
o Energy scheduled through GTAM contract shall be considered as deemed RPO compliance of the
buyer.
o Daily & Weekly Contracts – Bidding will take place on MWh basis.
o Price discovery will take place on a continuous basis i.e. price time priority basis.
• Significance and benefits of the move:
o It would lessen the burden on renewable energy-rich States and incentivize them to develop RE
capacity beyond their own RPO.
o It would promote RE merchant capacity addition and help in achieving RE capacity addition targets of
the country.
o It will also lead to increase in number of participants in renewable energy sector. It will benefit buyers
of RE through competitive prices and transparent and flexible procurement. It will also benefit RE
sellers by providing access to pan- India market.
Q 84.C
• ‘Status of Tigers, Co-predators & Prey in India’ report (2018)
• India recorded a 33% increase in tiger numbers from 2014 to 2018, according to the summary of the
report, All India Tiger Estimation Results, released last year. There were 2,967 tigers in 2018, compared
to 2,226 in 2014. India has 70 percent of world’s tiger population.
• According to the population estimation of tigers in reserves for 2018-19, Corbett has 231 tigers followed
by Nagarhole and Bandipore reserves in Karnataka with 127 and 126 tigers respectively. Assam’s
Kaziranga and Madhya Pradesh’s Bandhavgarh recorded 104 tigers each, the report said.
• Uttarakhand’s Corbett Tiger Reserve (CTR) has reported the highest tiger density among India’s 50
reserves with 14 tigers per 100 sq km, followed by Kaziranga, Nagarhole and Orang tiger reserves. Hence
option (c) is the correct answer.
• It said some reserves like Similipal (Odisha), Amrabad and Kawal in Telangana, Nagarjunsagar Sri
Sailam (Andhra Pradesh), Palamau (Jharkhand), Sanjay-Dubri (MP), Nameri and Manas in Assam, Buxa
(West Bengal), Dampa (Mizoram), Anshi Dandeli (Karnataka) and Pakke (Arunachal Pradesh) are below
their potential and require resources and targeted management.

Q 85.D
• Recent Context - On the eve of International Coastal Clean-Up Day, India has launched its own eco-label
‘BEAMS’ (Beach Environment & Aesthetics Management Services).
• On the lines of Blue Flag Certification, the Union Ministry of Environment, Forest and Climate
Change (MoEFCC) has launched India’s own eco – label ‘BEAMS’ (Beach Environment & Aesthetics
Management Services) under its Integrated Coastal Zone Management (ICZM) project.
34 www.visionias.in ©Vision IAS
• This is one of the several other projects of ICZM that Govt of India is undertaking for the sustainable
development of coastal regions, striving for globally recognized and the coveted eco-label ‘Blue flag.
• The objective of BEAMS program is to abate pollution in coastal waters, promote sustainable
development of beach facilities, protect & conserve coastal ecosystems & natural resources etc. This
program promotes beach recreation in absolute harmony with nature. Hence option (d) is the correct.
• Blue Flag Certification - It is one of the world’s most recognised voluntary eco-labels awarded to
beaches, marinas, and sustainable boating tourism operators. In order to qualify for the Blue Flag, a series
of stringent environmental, educational, safety, and accessibility criteria must be met and maintained. It is
operated by the Foundation for Environmental Education, headquartered in Copenhagen, Denmark. Blue
Flag beaches are considered to be the cleanest beaches in the world.
Q 86.C
• Recently, a nationwide citizen poll was organised by the National Butterfly Campaign Consortium (a collective
of 50 butterfly experts and enthusiasts) to choose the national butterfly, in which three species garnered the highest
number of votes - Krishna Peacock (Papilio krishna), Indian Jezebel (Delias eucharis), and Orange
Oakleaf (Kallima inachus), the frontrunners, have unique features such as ability to camouflage as a dead leaf,
exhibit iridescence to stave off predators, and aid farmers in getting rid of pests.
• The organisers submitted the list of the top-three to the Ministry of Environment, Forests and Climate
Change, the onus is on the Centre to choose one among them.
• Indian Jezebel, blessed with a vibrant colour pattern, including vermilion (haldi – kumkum) , the Indian
Jezebel (or Common Jezebel) is known to deter its predators with its flashy wing colours. Regarded as
soldiers of farmers, they also prey on parasites that infest fruit-bearing plants. Widely distributed, the
species can be spotted in gardens and other lightly wooded areas.
• Krishna Peacock, a flagship species for biodiversity and conservation , is generally found in large
numbers in the Himalayas. Possessing a peculiarly large swallowtail, its iridescent green scales diffract
light to coat itself in radiance.
• Orange Oakleaf is commonly known as ‘dead leaf’ for its ability to camouflage as a dry autumn leaf
while striking a stationary pose with its wings closed. The masquerade enables the species to prevent it
from being devoured by birds in the moist forests of northern Western Ghats, central, northern and
northeastern parts of India where they are generally found. Besides, the Oakleaf is also known to exhibit
polyphenism as it assumes specific colour and size during dry and wet seasons. Hence the correct
answer is option (c).
Q 87.D
• For centuries, beach sand has been mined for use as aggregate in concrete, for heavy minerals like
monazite, rare earth minerals, and for construction fill. The global extent and impact of this phenomenon
has gone relatively unnoticed by academics, NGOs, and major news sources.
• Sand mining in many regions has resulted in the complete destruction of beach (and related) ecosystems
along with severe impacts to coastal protection and tourism.
• Problems caused by beach sand mining include:
o Destruction of natural beaches and the ecosystems they protect (e.g. dunes, wetlands)
o Habitat loss for globally important species (e.g. turtles, shorebirds)
o Increased shoreline erosion rates
o Reduced protection from storms, tsunamis, and wave events
o Economic losses through tourist abandonment and loss of coastal aesthetics.
o Retreat of waterline by disrupting the critical balance of natural depositional and erosional
forces. The waterline is the line where the hull of a ship meets the surface of the water.
o Reducing the amount of sunlight and oxygen in the water due to turbidity created by removing
sediments from the sea and riverbeds,
o Causes land erosion and intrusion of seawater during high tide.
• The Mines and Minerals (Development and Regulation) Act, 1957 has empowered state governments to
make rules to prevent illegal mining, transportation and storage of minerals. Besides it, Government has
notified various regulations to control unabated sand mining in coastal states.
• Recently Union Ministry of Mines changed the rules that earlier allowed private companies to mine rare
earth minerals found in the beach sands.
Q 88.A
• Freshwater ecosystems can be divided into two categories:
o A lentic ecosystem entails a body of standing water, ranging from ditches, seeps, ponds, seasonal
pools, basin marshes, swamp and lakes. Deeper waters, such as lakes, may have layers of
ecosystems, influenced by light. Ponds, due to their having more light penetration, are able to support
a diverse range of water plants. These are also known as Pond ecosystem.

35 www.visionias.in ©Vision IAS


o A lotic ecosystem can be any kind of moving water, such as a run, creek, brook, river, spring,
channel or stream. The water in a lotic ecosystem, from source to mouth, must have atmospheric
gases, turbidity, longitudinal temperature gradation and material dissolved in it.
o Hence, option (a) is the correct answer.

Q 89.D
• Oil spills are always harmful, whether accidental or deliberate as it contains hydrocarbons which are
carcinogenic and also cause great damage to marine ecosystem disturbing its food chain and hence a
threat to entire marine community.
• Many chemical and physical methods are used to remove the spilled oil from ocean floor, each having
some drawbacks. Bioremediation has a promising future in oil spill removal in which indigenous or
exogenous microbes are used to clean the oil spill.
• These microbes are mostly fungi, bacteria, algae and yeasts, which are already present in the water.
• Many bacterias like A. borkumensis lives in all of the world’s oceans. Such bacterias have been
cultured to produce certain enzymes that break down crude oil and gas. A new technique of using the
bacteria to get rid of oil spills has been called “Oil Zapping".
• Though not in abundance in marine ecosystem, fungi are found to be better degrader of
hydrocarbon than other microbes, and its usage to clean up oil spills is burgeoning.
• Algae are widely used as biosorbent for the sorption of heavy metals and have also been found effective
for the absorption of diesel and lubricating oil. Algae can be used for development of low-cost sorbents
and can facilitate the removal of oil from contaminated water. Sorption is affected both by the type of oil
and type of algae.
• Hence, option (d) is the correct answer.

Q 90.A
• Evolutionarily Distinct and Globally Endangered (EDGE) species are threatened species that have
few or no close relatives. EDGE species are usually extremely distinct in the way they look, live and
behave as well as in their genetic make-up. If they disappear, there will be nothing like them left on the
planet. Hence statement 1 is correct.
• Every species in a particular taxonomic group (e.g. mammals or amphibians) is scored according to the
amount of unique evolutionary history it represents (Evolutionary Distinctiveness, or ED), and its
conservation status (Global Endangerment, or GE).
• Globally Endangered (GE) scores for each species are based on the IUCN Red List Categories (Critically
Endangered, Endangered, Vulnerable, Near Threatened and Least Concern). Species which are Critically
Endangered receive a higher score than less threatened species, which in turn, receive a higher score than
those not currently in danger of extinction.
• Species uniqueness’ can be measured as an 'Evolutionary Distinctiveness' (ED) score, using a phylogeny,
or evolutionary tree.
• The ED and GE scores are combined to produce an overall EDGE score for each species. EDGE scores
are calculated by multiplying ED and GE together. In mathematical terms, EDGE scores are an estimate
of the expected loss of evolutionary history per unit time.
• The Zoological Society of London launched the EDGE of Existence Programme in 2007 to raise
awareness and funds for the conservation of these species.
• There are around 100 mammals listed as edge species including Asiatic Elephants, Red Panda, Ganges
Dolphin, Dugong etc.
• Asiatic lions are not categorized as EDGE species. Hence statement 2 is not correct.

Q 91.B
• With the passing of the Wildlife Protection Act 1972, declaring a state animal or bird was a guideline
issued to the state government in the 1970s. Certain animals are endemic to certain parts of the country,
declaring them as state emblems garnered special attention for the people. It also gave the people of the
state a sense of pride in that species, a part of their natural heritage.
• State Animals, Birds, Trees and Flowers of India
State/UT Animal Bird Tree Flower
West Bengal Fishing Cat White-breasted Kingfisher Chatian Shephali
Uttarakhand Musk Deer Himalayan Monal Burans Brahma Kamal
Delhi Nilgai House Sparrow NA NA
• Hence the correct answer is option (b).
36 www.visionias.in ©Vision IAS
Q 92.C
• Biodiversity hot spots are the areas with a significant reservoir of biodiversity and the species richness is
threatened by human activities.
• ‘Cool-spots’ on the other hand are the world’s last refuges where high numbers of threatened
species still persist and where wildlife is still thriving. Cool-spots could be the result of protection or
because of intact habitat that has not been cleared yet.
• Some of the "cool spots" identified include parts of the Amazon rainforest, Andes Mountains, and tundra
and boreal forests of Russia and North America. The Amazon rainforest is the overwhelmingly
dominant global cool spot.
• Hence both statements are correct.
Q 93.C
• Each pond or lake has several different zones that divide the water column from top to bottom and side to
side. The zones are Littoral Zone, Limnetic Zone, Profundal Zone, Euphotic Zone, and Benthic Zone.
• Littoral Zone
o It is the shore area of the lake or pond. It is the topmost zone near the shores. It consists of the area
from the dry land sloping to the open water and can be very narrow or very wide.
o It is a shallow zone and is the warmest since it is the area that light hits contains flora such as rooted
and floating aquatic plants, and contains a very diverse community, which can include several species
of algae, grazing snails, clams, insects, crustaceans, fishes, and amphibians.
o It gets a lot of nutrients from runoff and non-point source pollution. Therefore, it typically has an
abundance of aquatic plant and algae growth.
• Limnetic Zone
o It is generally classified as the open water area of the lake or pond. This is a much larger section of
water in oligotrophic or younger ponds and lakes than it is in eutrophic or older bodies of water.
Within the limnetic zone are two separate sections.
o The upper portion of the limnetic zone near the surface of the water is the Euphotic Zone or
Epilimnion (warm water region). This is the portion of water that receives sunlight. The zone ends
where the sunlight fails to penetrate the water. The euphotic zone is where algae and other aquatic
plants thrive (along with the littoral zone). Also this is the typical area of dense fish populations
because oxygen levels are typically higher due to contact with the air.
o Below the euphotic zone is the Profundal Zone or Hypolimnion (cold water region). The profundal
zone is located below the thermocline where the sunlight does not penetrate. Again, the size of this
zone depends on the age and water clarity of the pond or lake. The profundal zone typically has lower
fish populations because of the lack of oxygen during many parts of the year.
• Benthic Zone
o It is the bottom of the pond or lake and consists of organic sediments and soil. The benthic zone is the
pond or lakes digestive system. This is where bacteria decompose organic matter from dead algae,
aquatic plants, and fish and animal waste. The more organic matter in the pond, the more
decomposition taking place. Decomposition can take place either aerobically (in the presence of
oxygen) or anaerobically (without oxygen). It is much better to have aerobic decomposition because it
is a faster process and the byproducts are easier to handle. The benthic zone increases as the pond or
lake ages.
Q 94.C
• Bioaccumulation is the intake of a chemical and its concentration in the organism by all possible means,
including contact, respiration and ingestion.
• Four types of variables affect bioaccumulation-physical-chemical properties of the contaminant
molecules, environmental conditions, characteristics of the exposed organism, and the organism's food
chain. These factors may act in concert or in opposition resulting in a range of bioaccumulation potentials.
• Physical-chemical properties of the contaminant molecule play a central role in the bioaccumulation
process. Features of chemicals that confer the tendency to bioaccumulate include
o lipophilicity or fat loving, which is directly related to the magnitude of a chemical's solubility in
octanol, and characterized by the magnitude of the octanol-water partition coefficient (kW)
o low water solubility or hydrophobicity due to the lack of polar functional groups. Hence, option 1 is
not correct.
o structural stability resulting in environmental persistence (years instead of days). Hence, option 3 is
correct.
o chemicals of moderate molecular weight and size (Le., molecular weight of about 350 and
molecular breadth of less than 10 Angstroms), and lacking ionizable functional groups have a greater
tendency to bioaccumulate. Hence, option 2 is not correct.
37 www.visionias.in ©Vision IAS
• Environmental Related Factors: Potential of a compound to bioaccumulate is affected by its
environmental stability or persistence. The effects of environmental degradation processes (e.g.,
hydrolysis, photolysis and microbial degradation) on contaminant molecules typically result in more
hydrophilic (water-loving) or polar products, which have lower bioaccumulation potentials than did the
parent compounds. Some exceptions to this outcome do exist, especially under conditions of low oxygen.
However, the overall effect of these degradation processes is to reduce parent compound concentrations
and organism exposure time, thereby decreasing the amounts of residues bioaccumulated.
• Organism Related Factors: Lipophilic contaminants are accumulated by aquatic organisms from the
water via respiration and from ingested food or sediments. Bioconcentration (uptake from water) is
generally viewed as the predominant route of uptake for most chemicals (including most PAHs) by
aquatic organisms. Because liters of water per day are ventilated across the gill membranes of fish, the gill
is generally the principal point of contaminant entry into an aquatic organism.
• Food Chain Related Factors: Biomagnification is the increase in the bioaccumulation factors (BAFs) of
certain chemicals in organisms occupying sequentially higher trophic positions in a food chain. This
phenomenon occurs because of the following sequence of events. As lipids of contaminated prey are
digested in the gut of predators, the capacity of the digestate (due to its increased polarity) to retain
nonmetabolized lipophilic contaminants is reduced, resulting in the net transfer of these chemicals to the
predator's lipid-rich tissues. Since the predator continues to consume numerous prey, the rates of uptake
by the diet can exceed the rate of elimination, resulting in contaminant concentrations higher than those
that would be found in the predator's fatty tissues at equilibrium. If this animal is, in turn, consumed by a
predator of higher trophic level, a further magnification in residue concentrations can occur. In cases
where the predators are fish-eating birds and mammals having high consumption rates of contaminated
fatty prey and limited elimination pathways, biomagnification can result in residue concentrations that are
100-fold higher than the equilibrium values.
Q 95.C
• NITI Aayog and Quality Council of India launched the ‘National Program and Project
Management Policy Framework’ (NPMPF), envisaged to bring radical reforms in the way
infrastructure projects are executed in India. This framework aims to formulate reforms in the way
large and mega infrastructure projects are managed in India, with an action plan to:
o Adopt a program and project management approach to infrastructure development.
o Institutionalize and promote the profession of program and project management and build a workforce
of such professionals,
o Enhance institutional capacity and capability of professionals
• Hence statements 1 and 2 are correct.

Q 96.D
• Heatwaves can also occur in the ocean and these are known as marine heatwaves or MHWs. MHW is a
short period of abnormally high temperatures in a sea or ocean.
• It is said to occur when sea surface temperatures in part of the ocean rise and stay above the
expected seasonal temperatures for at least five days in a row. Hence, statement 1 is correct.
• Marine heatwaves can occur during summer or winter in any ocean across the globe. They are
defined based on differences with expected temperatures for the location and time of year. Winter MHWs
are known as “winter warm-spells”. Hence, statement 2 is correct.
• Marine heatwaves can be caused by a whole range of factors, and not all factors are important for each
event. The most common drivers of marine heatwaves include ocean currents which can build up
areas of warm water and air-sea heat flux, or warming through the ocean surface from the atmosphere.
Winds can enhance or suppress the warming in a marine heatwave, and climate modes like El Niño can
change the likelihood of events occurring in certain regions. Hence, statement 3 is correct.
• A 2018 study showed that, since 1925, marine heatwaves have become 34 percent more frequent and they
are lasting longer. The majority of marine heatwaves, about 87 percent, can be attributed to human-caused
global warming.
• Some of the oceans' most fundamental ecosystems—coral reefs, beds of seagrass and kelp forests—are at
high risk from disturbance by marine heatwaves.
Q 97.B
• For any species, the minimal requirement is one more species on which it can feed. Even a plant species,
which makes its own food, cannot survive alone; it needs soil microbes to break down the organic matter
in soil and return the inorganic nutrients for absorption.
• Interspecific interactions arise from the interaction of populations of two different species. They could be
beneficial, detrimental or neutral (neither harm nor benefit) to one of the species or both.
38 www.visionias.in ©Vision IAS
o Both the species benefit in mutualism and both lose in competition in their interactions with each
other. Example: Lichens represent an intimate mutualistic relationship between a fungus and
photosynthesising algae or cyanobacteria.
o In both parasitism and predation only one species benefits (parasite and predator, respectively) and
the interaction is detrimental to the other species (host and prey, respectively). Example: Animals
eating plants.
o The interaction where one species is benefitted and the other is neither benefitted nor harmed is
called commensalism. Example: An orchid growing as an epiphyte on a mango branch. Hence the
correct answer is option (b).
o In amensalism, on the other hand, one species is harmed whereas the other is unaffected.
Example: In dense forests all around the world, amensalism takes place frequently as large trees tower
over shorter plants stealing most or all of the sunlight.
• Predation, parasitism and commensalism share a common characteristic– the interacting species live
closely together.

Q 98.C
• The Himalayan ranges show a succession of vegetation from the tropical to the tundra, which changes
in with the altitude.
• Deciduous forests are found in the foothills of the Himalayas. It is succeeded by the wet temperate
type of forests between an altitude of 1,000-2,000 m. In the higher hill ranges of northeastern India, hilly
areas of West Bengal and Uttaranchal, evergreen broadleaf trees such as oak and chestnut are
predominant.
• Between 1,500-1,750 m, pine forests are also well-developed in this zone, with Chir Pine as a very useful
commercial tree. Deodar, a highly valued endemic species grows mainly in the western part of the
Himalayan range. Deodar is a durable wood mainly used in construction activity. Similarly, the chinar
and the walnut, which sustain the famous Kashmir handicrafts, belong to this zone.
• Blue pine and spruce appear at altitudes of 2,225-3,048 m. At many places in this zone, temperate
grasslands are also found. But in the higher reaches, there is a transition to Alpine forests and
pastures. Silver firs, junipers, pines, birch and rhododendrons, etc. occur between 3,000-4,000 m.
However, these pastures are used extensively for transhumance by tribes like the Gujjars, the Bakarwals,
the Bhotiyas and the Gaddis.
• The southern slopes of the Himalayas carry a thicker vegetation cover because of relatively higher
precipitation than the drier north-facing slopes. At higher altitudes, mosses and lichens form part of the
tundra vegetation.
• Hence the correct order is 1-2-3-4.

Q 99.B
• Ecosystem services are the many and varied benefits to humans provided by the natural environment and
from healthy ecosystems. Such ecosystems include, for example, agroecosystems, forest ecosystems,
grassland ecosystems and aquatic ecosystems.
• The Millennium Ecosystem Assessment (MA), a major UN-sponsored effort to analyze the impact of
human actions on ecosystems and human well-being, identified four major categories of ecosystem
services: provisioning, regulating, cultural and supporting services.
• Provisioning Services: When people are asked to identify a service provided by nature, most think of
food. Fruits, vegetables, trees, fish, and livestock are available to us as direct products of ecosystems. A
provisioning service is any type of benefit to people that can be extracted from nature. Along with food,
other types of provisioning services include drinking water, timber, wood fuel, natural gas, oils, plants
that can be made into clothes and other materials, and medicinal benefits. Hence, pair 3 is not correctly
matched.
• Regulating Services: Ecosystems provide many of the basic services that make life possible for
people. Plants clean air and filter water, bacteria decompose wastes, bees pollinate flowers, and tree
roots hold soil in place to prevent erosion. All these processes work together to make ecosystems clean,
sustainable, functional, and resilient to change. A regulating service is the benefit provided by ecosystem
processes that moderate natural phenomena. Regulating services include pollination, decomposition,
water purification, erosion and flood control, and carbon storage and climate regulation. Hence, pair 2 is
correctly matched.
• Supporting Services: The natural world provides so many services, sometimes we overlook the most
fundamental. Ecosystems themselves couldn't be sustained without the consistency of underlying natural
processes, such as photosynthesis, nutrient cycling, the creation of soils, and the water cycle. These
39 www.visionias.in ©Vision IAS
processes allow the Earth to sustain basic life forms, let alone whole ecosystems and people. Without
supporting services, provisional, regulating, and cultural services wouldn't exist. Hence, pair 1 is not
correctly matched.
• Cultural Services: As we interact and alter nature, the natural world has in turn altered us. It has guided
our cultural, intellectual, and social development by being a constant force present in our lives. The
importance of ecosystems to the human mind can be traced back to the beginning of mankind with ancient
civilizations drawing pictures of animals, plants, and weather patterns on cave walls. A cultural service is
a non-material benefit that contributes to the development and cultural advancement of people, including
how ecosystems play a role in local, national, and global cultures; the building of knowledge and the
spreading of ideas; creativity born from interactions with nature (music, art, architecture); and recreation.

Q 100.B
• Recent Context - United States National Aeronautics and Space Administration (NASA) has
captured images depicting the wet and dry cycles of Etosha Pan, located in Namibia in Africa.
• Etosha Pan - It is a large endorheic salt pan forming part of the Kalahari Basin in the north of
Namibia. Hence option (b) is correct. It is hollow in the ground, wherein water may collect or in which a
deposit of salt remains after the water has evaporated. It is 120-kilometre-long dry lakebed. The pan is
mostly dry but after a heavy rain, it will acquire a thin layer of water which is heavily salted by the
mineral deposits on the surface.
• According to NASA Earth Observatory, the salt pan receives most rainfall — as much as 46 centimetres
— every year between October and March. During the dry season from April through September, water in
the basin evaporates — depositing salt and other minerals on the land.
• Its surroundings are protected as Etosha National Park which is Namibia’s second-largest wildlife park.
• It is also designated as a Ramsar wetland of international importance.

Upsc 4 EveryOne is initiative to provide watermarks and bookmarks free


pdfs to you. Share and Subscribe our telegram channel
@Upsc_4_EveryOne1

https://t.me/Upsc_4_EveryOne1

40 www.visionias.in ©Vision IAS

You might also like